Tag Archives: DapAn

Đề thi và đáp án chọn đội dự tuyển PTNK năm 2022

Thời gian làm bài 120 phút.

Bài 1. Cho $a, b, c \geq 0$ thỏa $a^2+b^2+c^2=1$. Tìm giá trị lớn nhất và giả trị nhỏ nhất của biểu thức $P=a b+b c+c a-2(a+b+c)$.

Bài 2. Cho $k, n \in Z^{+}$, có bao nhiêu đơn ánh từ $\{1, 2, \cdots, 2k+1\} \to \{1, 2, \cdots, 2n\}$ thỏa $f(1) < f(2) < \ldots < f(k) < f(k+1) > f(k+2)>\ldots> f(2 k)>f(2 k+1)$ và $f(k+1) \neq 2 n-2$.

Bài 3. Cho $n$ là số nguyên dương, kí hiệu $a(n)=1+2+\ldots+n$ và $b(n)=1^2+2^2+\ldots+n^2$. Hỏi có tồn tại số $n$ sao cho $2(n+1) a(n)+3 b(n)-1$ là số chính phương?

Bài 4. Cho tam giác $A B C$ có $2 A=5 B=10 C$. Phân giác trong $B D$ cẳt trung tuyển $C M$ tại I. Một đường thẳng $d$ đi qua $D$ vuông góc với $A C$ cắt $B C$ và $A I$ lần lượt tại $E$ và $K . A E$ cắt $C K$ tại $F$. Chứng minh: $M F$ song song $B K$.

Lời giải tham khảo

Bài 1. Đặt $t=a+b+c$ ta có $a(1-a) \geq 0, b(1-b) \geq 0, c(1-c) \geq$, suy ra $a+b+c \geq$ $a^2+b^2+c^2=1$, và $(a+b+c)^2 \leq 3\left(a^2+b^2+c^2\right)=3$, suy ra $a+b+c \leq \sqrt{3}$ Ta có $1=(a+b+c)^2-2(a b+b c+a c) \Rightarrow a b+b c+c a=\frac{t^2-1}{2}$.
Do đó $P=\frac{t^2-1}{2}-2 t=\frac{1}{2} t^2-2 t-\frac{1}{2}$ với $1 \leq t \leq \sqrt{3}$.
Khảo sát hàm bậc hai trong đoạn ta có $\max P=-2$ khi $t=1$ và $\min P=1-2 \sqrt{3}$.
Vậy $\max P=-2$ khi $a=1, b=c=0$ và min $P=1-2 \sqrt{3}$ khi $a=b=c=\frac{1}{\sqrt{3}}$.

Bài 2. Do đó $f$ là đơn ánh, $\operatorname{Im} f$ là một tập con có $2 k+1$ phần tử của $A$, mặt khác $f(k+1)$ là giá trị lớn nhất nên $\operatorname{Im} f$ có giá trị lớn nhất khác $2 n-2$.
Ta đếm số tập con có $2 k+1$ phần tử của $A$ mà phần tử lớn nhất khác $2 n-2$. Số tập con có $2 k+1$ của $A$ là $C_{2 n}^{2 k+1}$, số tập con có $2 k+1$ mà có phần tử lớn nhất $2 n-2$ là bằng với số tập con có $2 k$ phần tử của ${1,2, \cdots 2 k-3}$, là $C_{2 n-3}^{2 k}$.
Do đó theo nguyên lí bù trừ số tập con có $2 k+1$ của tập $A$ mà phần tử lớn nhất khác $2 n-2$ là $\left(C_{2 n}^{2 k+1}-C_{2 n-3}^{2 k}\right)$.
Tiếp theo ta đếm số đơn ánh từ ${1,2, \cdots, 2 k+1}$ tới $A^{\prime}=\left\{a_1, a_2, \cdots, a_{2 k+1}\right\}$ thỏa đề bài, ta có $f(k+1)=a_{2 k+1}$, nên số đơn ánh bằng số cách chọn $k$ phần tử từ $A^{\prime}$ nên bằng $C_{2 k}^k$.
Vậy số đơn ánh thỏa đề bài $C_{2 k}^k\left(C_{2 n}^{2 k+1}-C_{2 n-3}^{2 k}\right)$

Bài 3. Ta có $a(n)=\frac{n(n+1)}{2}, b(n)=\frac{n(n+1)(2 n+1)}{6}$
Khi đó $P(n)=2(n+1) a(n)+3 b(n)-1=\frac{n(n+1)(4 n+3)}{2}-1$.
Giả sử $P(n)$ là số chính phương ta có $n(n+1)(4 n+3)=2\left(x^2+1\right)$, ta có $n(n+1)(4 n+3)$ luôn có ước nguyên tố dạng $p=4 k+3$, suy ra $p \mid 2\left(x^2+1\right)$ suy ra $p|x, p| 1$, vô lí! Vậy không tồn tại $n$ để $P(n)$ là số chính phương.

Bài 4.

Ta tính được $\angle A=\frac{5 \pi}{8}, \angle B=\frac{\pi}{4}, \angle C=\frac{\pi}{8}$. Vẽ đường cao $A N, N$ thuộc $B C$.
Ta có $\frac{B N}{N C}=\frac{A N}{N C}=\frac{\sin C}{\cos C}$ và $\frac{A D}{C D}=\frac{A B}{B C}=\frac{\sin C}{\sin 5 C}, \sin 5 C=\cos C$, suy ra $\frac{B N}{N C}=\frac{A D}{C D}$, do đó $A N, B D, C M$ đồng quy tại $I$ và $D N | A B$.
Ta có $\angle B A N=\angle A N D=\angle A C K=2 \angle A C K$, suy ra $A C K$ cân và $N$ là trung điểm $A K$, từ đó tam giác $A B K$ vuông cân.
Khi đó $\angle F N K=\angle A C K=45^{\circ}=\angle A K B$ và $\angle A N M=45^{\circ}$, do đó $M, N, F$ thẳng hàng và $M F | B K$.

Đề thi Học sinh giỏi Quốc gia năm 2021 (VMO 2021)

Ngày thi thứ nhất. Thời gian làm bài 180 phút.

Bài 1 (5 điểm). Cho dãy số thực $\left(x_n\right)$ có $x_1 \in\left(0, \frac{1}{2}\right)$ và $x_{n+1}=3 x_n^2-2 n x_n^3$ với mọi $n \geq 1$.
a) Chứng minh $\lim x_n=0$.
b) Với mỗi $n \geq 1$ đặt $y_n=x_1+2 x_2+\cdots+n x_n$. Chứng minh rằng dãy $\left(y_n\right)$ có giới hạn hữu hạn.

Bài 2 (5 điểm). Tìm tất cả các hàm số $f: \mathbb{R} \rightarrow \mathbb{R}$ thỏa mãn
$$
f(x) f(y)=f(x y-1)+x f(y)+y f(x)
$$
với mọi số thực $x, y$.
Bài 3 (5 điểm). Cho tam giác nhọn không cân $A B C$ có trực tâm $H$ và $D, E, F$ lần lượt là chân đường cao hạ từ các đỉnh $A, B, C$. Gọi $(I)$ là đường tròn ngoại tiếp tam giác $H E F$ với tâm $I$ và $K, J$ lần lượt là trung điểm $B C, E F$. Cho $H J$ cắt lại $(I)$ tại $G$, $G K$ cắt lại $(I)$ tại $L$.
a) Chứng minh rằng $A L$ vuông góc với $E F$.
b) Cho $A L$ cắt $E F$ tại $M, I M$ cắt lại đường tròn ngoại tiếp tam giác $I E F$ tại $N$, $D N$ cắt $A B, A C$ lần lượt tại $P, Q$. Chứng minh rằng $P E, Q F, A K$ dồng quy.

Bài 4(5 điểm). Với số nguyên $n \geq 2$, gọi $s(n)$ là tổng các số nguyên dương không vượt quá $n$ và không nguyên tố cùng nhau với $n$.
a) Chứng minh $s(n)=\frac{n}{2}(n+1-\varphi(n))$, trong đó $\varphi(n)$ là số các số nguyên dương không vượt quá $n$ và nguyên tố cùng nhau với $n$.
b) Chứng minh rằng không tồn tại số nguyên $n \geq 2$ thỏa mãn $s(n)=s(n+2021)$.

Ngày thi thứ 2. Thời gian làm bài 180 phút.

Bài 5 (6 điểm). Cho đa thức $P(x)=a_{21} x^{21}+a_{20} x^{20}+\cdots+a_1 x+a_0$ có các hệ số thuộc $[1011,2021]$. Biết rằng $P(x)$ có nghiệm nguyên và $c$ là một số dương sao cho $\left|a_{k+2}-a_k\right| \leq c$ với mọi $k \in{0,1, \ldots, 19}$.
a) Chứng minh rằng $P(x)$ có đúng một nghiệm nguyên.
b) Chứng minh $\sum_{k=0}^{10}\left(a_{2 k+1}-a_{2 k}\right)^2 \leq 440 c^2$.

Bài 6 (7 điểm). Một học sinh chia tất cả 30 viên bi vào 5 cái hộp được đánh số $1,2,3,4,5$ (sau khi chia có thể có hộp không có viên bi nào).
a) Hỏi có bao nhiêu cách chia các viên bi vào các hộp (hai cách chia là khác nhau nếu có một hộp có số bi trong hai cách chia là khác nhau)?
b) Sau khi chia, học sinh này sơn 30 viên bi đó bởi một số màu (mỗi viên được sơn đúng một màu, một màu có thể sơn cho nhiều viên bi), sao cho không có 2 viên bi nào trong cùng một hộp có màu giống nhau và từ hai hộp bất kì không thể chọn ra được 8 viên bi được sơn bởi 4 màu. Chứng minh rằng với mọi cách chia, học sinh đều phải dùng không ít hơn 10 màu để sơn bi.
c) Hãy chỉ ra một cách chia sao cho với đúng 10 màu, học sinh có thể sơn bi thỏa mãn các điều kiện ở câu b).

Bài 7 (7 điểm). Cho tam giác nhọn không cân $A B C$ nội tiếp đường tròn $(O)$. Gọi $D$ là giao điểm hai tiếp tuyến của $(O)$ tại $B$ và $C$. Đường tròn đi qua $A$ và tiếp xúc với $B C$ tại $B$ cắt trung tuyến đi qua $A$ của tam giác $A B C$ tại $G$. Cho $B G, C G$ lần lượt cắt $C D, B D$ tại $E, F$.
a) Đường thẳng đi qua trung điểm của $B E$ và $C F$ lần lượt cắt $B F, C E$ tại $M, N$. Chứng minh rằng các điểm $A, D, M, N$ cùng thuộc một đường tròn.
b) Cho $A D, A G$ lần lượt cắt lại đường tròn ngoại tiếp các tam giác $D B C, G B C$ tại $H, K$. Trung trực của $H K, H E, H F$ lần lượt cắt $B C, C A, A B$ tại $R, P, Q$. Chứng minh rằng các điểm $R, P, Q$ thẳng hàng.

Lời giải tham khảo

Đề thi và đáp án học sinh giỏi quốc gia năm 2022 (VMO 2022)

Ngày thi thứ nhất. Thời gian làm bài 180 phút.

Bài 1 (5,0 điểm)
Cho $a$ là một số thực không âm và dãy số $\left(u_n\right)$ được xác định bởi
$$
u_1=6, u_{n+1}=\dfrac{2 n+a}{n}+\sqrt{\dfrac{n+a}{n} u_n+4}, \quad \forall n \geq 1 .
$$
a) Với $a=0$, chứng minh rằng $\left(u_n\right)$ có giới hạn hữu hạn và tìm giới hạn đó.
b) Với mọi $a \geq 0$, chứng minh rằng $\left(u_n\right)$ có giới hạn hữu hạn.

Bài 2 (5,0 điểm)
Tìm tất cả các hàm số $f:(0 ;+\infty) \rightarrow(0 ;+\infty)$ thoả mãn
$$
f\left(\dfrac{f(x)}{x}+y\right)=1+f(y), \forall x, y \in(0 ;+\infty) .
$$

Bài 3(5,0$ điểm)
Cho tam giác nhọn $A B C$. Các điểm $E, F$ lần lượt thay đổi trên tia đối của các tia $B A, C A$ sao cho $B F=C E(E \neq B, F \neq C)$. Gọi $M, N$ tương ứng là trung điểm của $B E, C F$ và $D$ là giao điểm của $B F$ với $C E$.
a) Gọi $I, J$ lần lượt là tâm đường tròn ngoại tiếp các tam giác $D B E, D C F$. Chứng minh rằng $M N$ song song với $I J$.
b) Gọi $K$ là trung điểm của $M N$ và $H$ là trực tâm của tam giác $A E F$. Chứng minh rằng $H K$ luôn đi qua một điểm cố định.

Bài 4 (5,0 điểm)
Với mỗi cặp số nguyên dương $(n, m)$ thoả mãn $n<m$, gọi $s(n, m)$ là số các số nguyên dương thuộc đoạn $[n ; m]$ và nguyên tố cùng nhau với $m$. Tìm tất cả các số nguyên dương $m \geq 2$ thoả mãn đồng thời hai điều kiện sau:
i) $\dfrac{s(n, m)}{m-n} \geq \frac{s(1, m)}{m}$ với mọi $n=1,2, \ldots, m-1$;
ii) $2022^m+1$ chia hết cho $m^2$.

Ngày thi thứ hai. Thời gian làm bài 180 phút.

Bài 5(6,0 điểm)
Cho $P(x)$ và $Q(x)$ là hai đa thức khác hằng, có hệ số là các số nguyên không âm, trong đó các hệ số của $P(x)$ đều không vượt quá 2021 và $Q(x)$ có ít nhất một hệ số lớn hơn 2021. Giả sử $P(2022)=Q(2022)$ và $P(x), Q(x)$ có chung nghiệm hữu tỷ $\dfrac{p}{q} \neq 0(p, q \in \mathbb{Z} ; p$ và $q$ nguyên tố cùng nhau). Chứng minh rằng $|p|+n|q| \leq Q(n)-P(n)$ với mọi $n=1,2, \ldots, 2021$.

Bài 6 (7,0 điểm)
Gieo 4 con súc sắc cân đối, đồng chất. Ký hiệu $x_i\left(1 \leq x_i \leq 6\right)$ là số chấm trên mặt xuất hiện của con súc sắc thứ $i(i=1,2,3,4)$.
a) Tính số các bộ $\left(x_1, x_2, x_3, x_4\right)$ có thể có.
b) Tính xác suất để có một số trong $x_1, x_2, x_3, x_4$ bằng tổng của ba số còn lại.
c) Tính xác suất để có thể chia $x_1, x_2, x_3, x_4$ thành hai nhóm có tổng bằng nhau.

Bài 7 (7,0 điểm)
Cho tam giác $A B C$ có $B, C$ cố định trên đường tròn $(O)$ ( $B C$ không đi qua tâm $O$ ) và điểm $A$ thay đổi trên cung lớn $\overparen{B C}$ sao cho $A B \neq A C$. Đường tròn nội tiếp $(I)$ của tam giác $A B C$ tiếp xúc với $B C$ tại $D$. Gọi $I_a$ là tâm đường tròn bàng tiếp góc $\widehat{B A C}, L$ là giao điểm của $I_a D$ với $O I$ và $E$ là điểm trên $(I)$ sao cho $D E$ song song với $A I$.
a) Đường thẳng $L E$ cắt đường thẳng $A I$ tại $F$. Chứng minh rằng $A F=A I$.
b) Trên đường tròn $(J)$ ngoại tiếp tam giác $I_a B C$ lấy điểm $M$ sao cho $I_a M$ song song với $A D, M D$ cắt lại $(J)$ tại $N$. Chứng minh rằng trung điểm $T$ của $M N$ luôn thuộc một đường tròn cố định.

Đáp án chính thức

(Nguồn: Bộ giáo dục Việt Nam)

Đề thi và đáp án kì thi chọn đội tuyển thi Quốc gia trường Phổ thông Năng khiếu năm học 2018 – 2019

ĐỀ THI

Ngày thi thứ nhất

Bài 1. Cho số nguyên $a>1$. Tìm giá trị lớn nhất của số thực $d$ sao cho tồn tại một cấp số cộng có công sai $d$, số hạng đầu tiên là $a$ và có đúng hai trong các số $a^2, a^3, a^4, a^5$ là những số hạng của cấp số cộng đó.

Bài 2. Cho $n$ số thực $x_1, x_2, \ldots, x_n$. Với mỗi $i \in{1,2, \ldots, n}$, gọi $a_i$ là số các chỉ số $j$ mà $\left|x_i-x_j\right| \leq 1$ và $b_i$ là số các chỉ số $j$ mà $\left|x_i-x_j\right| \leq 2$ ( $i$ và $j$ có thể bằng).

(a) Chứng minh rằng tồn tại $i$ để $b_i \leq 3 a_i$.

(b) Gọi $A$ là số cặp $(i, j)$ có thứ tự mà $\left|x_i-x_j\right| \leq 1$ và $B$ là số cặp $(i, j)$ có thứ tự mà $\left|x_i-x_j\right| \leq 2$ ( $i$ và $j$ có thể bằng nhau). Chứng minh rằng $B \leq 3 A$.

Bài 3. Cho $p$ là số tự nhiên. Xét phương trình nghiệm nguyên $x^3+x+p=y^2$.

(a) Tìm số nguyên tố $p$ nhỏ nhất dạng $4 k+1$ sao cho phương trình có nghiệm.

(b) Chứng minh rằng nếu $p$ là số chính phương thì phương trình trên có nghiệm nguyên dương.

Bài 4. Cho tam giác $A B C$ nhọn nội tiếp đường tròn $(O)$ với $B, C$ cố định và $A$ di động trên $(O)$. $D$ là trung điểm $B C$. Trên $A B$ lấy các điểm $M, P$ và trên $A C$ lấy các điểm $N, Q$ sao cho $D A=D P=D Q$, dồng thời $D M \perp A C, D N \perp A B$.

(a) Chứng minh rằng các điểm $M, N, P, Q$ cùng thuộc một đường tròn $(\mathcal{C})$ và $(\mathcal{C})$ luôn đi qua một điểm cố định.

(b) Chứng minh rằng tâm của $(\mathcal{C})$ luôn thuộc một đường tròn cố định.

 

Ngày thi thứ hai

Bài 5. Cho số thực $a \neq 0$. Tìm giới hạn của dãy số $\left(u_n\right)$ thoả mãn:

$\quad\quad\quad\quad\quad\quad\quad\quad u_1=0, u_{n+1}\left(u_n+a\right)=a+1, \forall n \in \mathbb{N}^*$

Bài 6. Tìm tất cả các hàm số $f: \mathbb{R}^{+} \rightarrow \mathbb{R}^{+}$thoả mãn điều kiện:

$\quad\quad\quad\quad f\left(x f\left(y^2\right)-y f\left(x^2\right)\right)=(y-x) f(x y) \forall x, y \in \mathbb{R}^{+}, x>y$

Bài 7. Cho $n=2018.2019$. Gọi $A$ là tập hợp các bộ $\left(a_1, a_2, \ldots, a_n\right)$ có thứ tự thoả mãn điều kiện $a_i \in{0,1} \forall i \in{1,2, \ldots, n}$ và $\sum_{i=1}^n a_i=2018^2$. Có bao nhiêu bộ $\left(a_1, a_2, \ldots, a_n\right)$ từ $A$ để $\sum_{i=1}^k a_i \geq \frac{a}{2}$ và $\sum_{i=n-k+1}^n a_i \geq \frac{k}{2} \forall k \in{1,2, \ldots, n}$ ?

Bài 8. Đường tròn $(\mathcal{C})$ tâm $I$ nội tiếp tam giác $A B C$ và tiếp xúc với các cạnh $A B, A C$ tại $E, F$. $A M, A N$ là các đường phân giác trong, phân giác ngoài của góc $\angle B A C(M, N$ nằm trên $B C)$. Gọi $d_M, d_N$ lần lượt là các tiếp tuyến của $(\mathcal{C})$ qua $M, N$ và khác $B C$.

(a) Chứng minh rằng $d_M, d_N, E F$ đồng quy tại điểm $D$.

(b) Lấy trên $A B, A C$ các điểm $P, Q$ thoả mãn $D P|A C, D Q| A B$. Gọi $R, S$ là trung điểm của $D E, D F$. Chứng minh rằng $I$ thuộc đường thẳng qua các trực tâm của hai tam giác $D P S, D Q R$.

 

LỜI GIẢI

Ngày thi thứ nhất

Bài 1. Cho số nguyên $a>1$. Tìm giá trị lớn nhất của số thực $d$ sao cho tồn tại một cấp số cộng có công sai $d$, số hạng đầu tiên là $a$ và có đúng hai trong các số $a^2, a^3, a^4, a^5$ là những số hạng của cấp số cônng đó.

Lời giải: Trước hết, ta chứng minh rằng $d=a^3-a$ thoả mãn điều kiện. Thật vậy, xét cấp số cộng có số hạng đầu là $a$ và công sai là $d=a^3-a$ thì

$\quad\quad\quad\quad\quad\quad\quad\quad\quad\quad \left\{\begin{array}{l}a^3=a+\left(a^3-a\right) \\ a^5=a+\left(a^3-a\right)\left(a^2+1\right)\end{array} .\right.$

Do đó $a^3, a^5$ cùng thuộc cấp số cộng có công sai $d=a^3-a$.

Giả sử rằng tồn tại giá trị $d>a^3-a$ thoả mãn điều kiện bài toán. Khi đó:

$\quad\quad\quad\quad\quad\quad\quad\quad\quad\quad a+d>a+a^3-a=a^3$

Dẫn đến hai số hạng thuộc cấp số cộng phải là $a^4$ và $a^5$. Lại để ý rằng $a>1$ nên có $a<a^4<a^5$, kết hợp lại thì phải tồn tại hai số nguyên dương $k<l$ sao cho:

$\quad\quad\quad\quad\quad\quad\quad\quad\quad\quad \left\{\begin{array}{l}a^4=a+k d \\ a^5=a+l d\end{array}\right.$

Từ đó $a(a+k d)=a+l d$ hay $d(l-a k)=a^2-a>0$.

Chú ý rằng ta có $d>0$ nên $l-a k>0$, hơn nữa $l-a k \in \mathbb{Z}$ nên $l-a k \geq 1$. Điều này dẫn đến $a^2-a \geq d>a^3-a$, vô lý do $a>1$.

Vậy giá trị lớn nhất của $d$ là $\max d=a^3-a$.

Bài 2. Cho $n$ số thực $x_1, x_2, \ldots, x_n$. Với mỗi $i \in{1,2, \ldots, n}$, gọi $a_i$ là số các chỉ số $j$ mà $\left|x_i-x_j\right| \leq 1$ và $b_i$ là số các chỉ số $j$ mà $\left|x_i-x_j\right| \leq 2(i$ và $j$ có thể bằng nhau).

(a) Chứng minh rằng tồn tại $i$ dể $b_i \leq 3 a_i$.

(b) Gọi $A$ là số cặp $(i, j)$ có thứ tự mà $\left|x_i-x_j\right| \leq 1$ và $B$ là số cặp $(i, j)$ có thứ tự mà $\left|x_i-x_j\right| \leq 2$ ( $i$ và $j$ có thể bằng nhau). Chứng minh rằng $B \leq 3 A$.

Lời giải . (a) Không mất tính tổng quát, giả sử $x_1 \leq x_2 \leq \ldots \leq x_n$.

Xét $k=\max [a_1, a_2, \ldots, a_n]$ và $a_i=k$, khi đó tồn tại $k$ số trong dãy là:

$\quad\quad\quad\quad x_u \leq x_{u+1} \leq \ldots \leq x_i \leq \ldots \leq x_v \text { với }\left|x_u-x_i\right|,\left|x_v-x_i\right| \leq 1 .$

Ngoài ra vì tính lớn nhất của $k$ nên $\left|x_{u-1}-x_i\right|>1,\left|x_{v+1}-x_i\right|>1$.

Trong $\left[x_u, x_v\right]$, có đúng $k$ chỉ số $j$ để $\left|x_j-x_i\right| \leq 1<2$. Còn trước $x_u$, xét hai số $x_r, x_s$ sao cho $x_r \leq x_s$ và $\left|x_r-x_i\right| \leq 2,\left|x_s-x_i\right| \leq 2$ thì:

$\quad\quad\quad\quad \left|x_r-x_s\right|=x_s-x_r=\left(x_i-x_r\right)-\left(x_i-x_s\right)<2-1=1$

nên sẽ có không quá $k$ số $j$ để $\left|x_j-x_i\right| \leq 2$ vì nếu ngược lại, sẽ có nhiều hơn $k$ số liên tiếp trong dãy cách nhau không quá 1 đơn vị, mâu thuẫn với tính lớn nhất của $k$. Tương tự với các số sau $x_v$, vì thế nên $b_i \leq 3 k$ kéo theo $b_i \leq 3 a_i$.

(b) Ta sẽ chứng minh bằng quy nạp theo $n$.

Với $n=1$ rõ ràng $A=B=1$ nên khẳng định hiển nhiên đúng. Giả sử kết quả đúng với $n \geq 1$, ta sẽ chứng minh nó cũng đúng với $n+1$.

Xét dãy số thực $T=\left(x_1, x_2, \ldots, x_{n+1}\right)$ bất kỳ và giả sử $x_1 \leq x_2 \leq \ldots \leq x_{n+1}$. Ký hiệu $A_T, B_T$ là số cặp có thứ tự các chỉ số $(i, j)$ tương ứng với định nghĩa của đề bài. Giả sử $k \geq 1$ là số lượng lớn nhất các số của $T$ được chứa trong một đoạn độ dài bằng 2 nào đó.

Gọi $x_i$ là số cuối cùng của dãy mà trong đoạn $\left[x_i-1, x_i+1\right]$ có chứa đúng $k$ số (kể cả $x_i$ ). Gọi $T^{\prime}$ là dãy mới sau khi bỏ $x_i$ đi. Khi đó, số lượng các số thuộc $T^{\prime}$ có trong $\left[x_i-1, x_i+1\right]$ là $k-1$, ngoài ra $x_i$ đã bị bỏ đi thuộc về đúng $2 k-1$ cặp của $A_T$.

Do đó: $A_T=A_{T^{\prime}}+2 k-1$.

Ta viết lại như sau

$\quad\quad\quad\quad \left[x_i-2 ; x_i+2\right]=\left[x_i-2 ; x_i-1\right] \cup\left[x_i-1 ; x_i+1\right] \cup\left[x_i+1 ; x_i+2\right]$

Trừ đoạn ở giữa thì hai đoạn đầu và cuối chứa tối đa $k$ phần tử của $T$. Hơn nữa, do định nghĩa số $x_i$ nên trong đoạn $\left[x_i+1 ; x_i+2\right]$ có tối đa $k-1$ phần tử của $T$. Từ đó có tối đa:

$\quad\quad\quad\quad\quad\quad\quad\quad\quad\quad\quad\quad 2(k-1)+k=3 k-2$

phần tử của $T$ (không tính $x_i$ ) thuộc $\left[x_i-2 ; x_i+2\right]$. Dẫn đến:

$\quad\quad\quad\quad\quad\quad B_T \leq 2(3 k-2)+1+B_{T^{\prime}}=3(2 k-1)+B_{T^{\prime}}$

Áp dụng giả thiết quy nạp, ta có $B_{T^{\prime}}<3 A_{T^{\prime}}$ nên từ các điều trên thì:

$\quad\quad\quad\quad B_T \leq 3(2 k-1)+B_{T^{\prime}}<3(2 k-1)+3 A_{T^{\prime}}=3\left(A_{T^{\prime}}+2 k-1\right)=3 A_T .$

Theo nguyên lý quy nạp, bài toán cũng đúng với $n+1$.

Vậy bài toán được chứng minh hoàn toàn.

Nhận xét. Bài toán này thật ra liên quan đến phương pháp xác suất trong tổ hợp, có thể xem tại quyển “The Probabilistic Method” của GS. Noga Alon. Ta xét một lời giải khác như sau:

(a) Chọn $i$ sao cho số các chỉ số $j$ để $\left|x_i-x_j\right| \leq 1$ là lớn nhất. Khi đó, số lượng chỉ số $j$ sao cho $x_j \in\left(x_i+1, x_i+2\right]$ tối đa là $a_i$, vì nếu không thì tồn tại $j$ để $a_j>a_i$. Tương tự, số lượng chỉ số $j$ sao cho $x_j \in\left[x_i-2, x_i-1\right)$ tối đa là $a_i$.

Chú ý rằng với các chỉ số $j$ để $\left|x_i-x_j\right| \leq 2$ thì ta có điều sau:

$\quad\quad\quad\quad x_j \in\left[x_i-2, x_i-1\right) \cup\left(x_i-1, x_i+1\right) \cup\left(x_i+1, x_i+2\right]$

Số lượng các chỉ số đó chính là $b_i$, dẫn đến $b_i \leq a_i+a_i+a_i=3 a_i$. Hơn nữa, nếu đẳng thức xảy ra, ta phải có mỗi đoạn (hay nửa khoảng) ở phân hoạch trên chứa chính xác $a_i$ chỉ số $j$ của $x_j$.

(b) Bài toán hiển nhiên đúng với $n=1$. Giả sử rằng tồn tại $n>1$ để kết luận không đúng, ta chọn $n$ nhỏ nhất. Ta cũng chọn $i$ sao cho $a_i$ lớn nhất.

Gọi $A^{\prime}, B^{\prime}$ tương ứng là số cặp chỉ số $(k, l)$ mà $\left|x_k-x_l\right| \leq 1$ và $\left|x_k-x_l\right| \leq 2$, trong đó $1 \leq k, l \leq n$ và $k, l \neq i$. Vì $n$ là phản ví dụ nhỏ nhất nên $B^{\prime} \leq 3 A^{\prime}$.

Các cặp chỉ số $(k, l)$ mà $k=i$ hoặc $l=i$ và $\left|x_k-x_l\right| \leq 1$ đều phải có dạng $(k, i)$ hoặc $(i, k)$ trong đó $k \neq i$ và $(i, i)$. Có tổng cộng $2\left(a_i-1\right)+1$ cặp như thế nên $A=A^{\prime}+2\left(a_i-1\right)+1$.

Tương tự thì $B=B^{\prime}+2\left(b_i-1\right)+1$. Do đó nếu $b_i \leq 3 a_i-1$ thì:

$\quad\quad\quad\quad B=B^{\prime}+2 b_i-1 \leq 3 A^{\prime}+2\left(3 a_i-1\right)-1=3\left(A^{\prime}+2 a_i-1\right)=3 A$

Điều này trái với việc $n$ là phản ví dụ nhỏ nhất. Do đó $b_i \geq 3 a_i$. Theo ý (a) thì $b_i \leq 3 a_i$, từ đây phải có $b_i=3 a_i$. Hơn nữa, số lượng chỉ số $j$ để thỏa mãn $x_j \in\left[x_i-2, x_i-1\right)$ hoặc $x_j \in\left(x_i+1, x_i+2\right]$ dều phải bằng $a_i$.

Với mỗi $j, j^{\prime}$ sao cho $x_j, x_{j^{\prime}} \in\left[x_i-2, x_i-1\right)$, ta có $\left|x_j-x_{j^{\prime}}\right|<1$, dẫn đến $a_j \geq a_i$. Mặt khác $a_i$ là lớn nhất có thể nên $a_j=a_i$. Tương tự, với mỗi $j$ sao cho $x_j \in\left(x_i+1, x_i+2\right]$ thì $a_j=a_i$. Như vậy với mọi $j$ sao cho $1<\left|x_i-x_j\right| \leq 2$ thì $a_j=a_i$. Cũng với cách chọn chỉ số $j$ đó, lập luận tương tự như những ý trên, ta cũng phải có $b_j=3 a_j$.

Xây dựng đồ thị $\mathcal{G}$ với các đỉnh được đánh số là $1,2, \ldots, n$ sao cho cặp đỉnh $(k, l)$ kề nhau khi và chỉ khi $1<\left|x_k-x_l\right| \leq 2$. Những lập luận trên cho thấy mọi đỉnh $j$ mà tồn tại một đường đi từ $i$ đến $j$ đều phải thỏa mãn $a_j=a_i$ và $b_j=3 a_j$. Gọi $\mathcal{X}$ là tập hợp tất cả các đỉnh $j$ sao cho tồn tại một đường đi từ $i$ dến $j$ trong $\mathcal{G}$. Đặt $\mathcal{Y}={1,2, \ldots, n} \backslash \mathcal{X}(\mathcal{Y}$ có thể rỗng $)$.

Bây giờ, gọi $A_y, B_y$ tương ứng là số cặp chỉ số $(k, l)$ có tính thứ tự, có thể bằng nhau mà $\left|x_k-x_l\right| \leq 1$ và $\left|x_k-x_l\right| \leq 2$, trong đó $k, l \in \mathcal{Y}$. Chú ý rằng $A_{\mathcal{Y}}=B_{\mathcal{Y}}=0$ nếu $\mathcal{Y}=\emptyset$. Bởi $n$ là phản ví dụ nhỏ nhất, ta phải có $B_{\mathcal{Y}} \leq 3 A_{\mathcal{Y}}$. Ta gọi $a_{y, k}$ và $b_{y, k}$ tương ứng là số chỉ số $j \in \mathcal{Y}$ mà $\left|x_j-x_k\right| \leq 1$ và $\left|x_j-x_k\right| \leq 2$. Định nghĩa tương tự $a_{\mathcal{X}, k}$ và $b_{\mathcal{X}, k}$.

Với mọi $k \in \mathcal{Y}$, dễ thấy $k$ không kề bất cứ đỉnh nào trong $\mathcal{X}$, vì vậy ta có được $b_{\mathcal{X}, k}=0$ và $b_k=b_{\mathcal{Y}, k}+a_{\mathcal{X}, k}$. Từ đây dẫn đến đẳng thức sau:

$\quad\quad\quad\quad\quad\quad B=\sum_{k \in \mathcal{X}} b_k+\sum_{k \in \mathcal{Y}} b_k=3 \sum_{k \in \mathcal{X}} a_k+\sum_{k \in \mathcal{Y}}\left(b_{y, k}+a_{\mathcal{X}, k}\right)$

Ta đồng thời có $\sum_{k \in \mathcal{Y}} b_{y, k}=B_{\mathcal{Y}} \leq 3 A_{\mathcal{Y}}$. Hơn nữa, ta cũng có được:

$\quad\quad A=\sum_{k \in \mathcal{X}} a_k+\sum_{k \in \mathcal{Y}} a_k=\sum_{k \in \mathcal{X}} a_k+\sum_{k \in \mathcal{Y}}\left(a_{\mathcal{Y}, k}+a_{\mathcal{X}, k}\right)=\sum_{k \in \mathbb{X}} a_k+A_{\mathcal{Y}}+\sum_{k \in \mathcal{Y}} a_{\mathcal{X}, k}$

Do đó:

$\quad\quad\quad\quad B \leq 3 A_{\mathcal{Y}}+\sum_{k \in \mathcal{Y}} a_{\mathcal{X}, k}+3 \sum_{k \in \mathcal{X}} a_k \leq 3\left(A_{\mathcal{Y}}+\sum_{k \in \mathcal{Y}} a_{\mathcal{X}, k}+\sum_{k \in \mathcal{X}} a_k\right)=3 A$

Điều này dẫn đến giả sử phản chứng là sai.

Vì vậy, với mọi số nguyên dương $n$, ta phải có $B \leq 3 A$. Bài toán kết thúc.

Bài 3. Cho $p$ là số tự nhiên. Xét phương trình nghiệm nguyên

$\quad\quad\quad\quad\quad\quad\quad\quad\quad\quad\quad\quad x^3+x+p=y^2 .$

(a) Tìm số nguyên tố $p$ nhỏ nhất dạng $4 k+1$ sao cho phương trình có nghiệm.

(b) Chứng minh rằng nếu $p$ là số chính phương thì phương trình trên có nghiệm nguyên dương.

Lời giải. (a) Các số nguyên tố có dạng $4 k+1$ là $5,13,17, \ldots$

Trước hết, ta thấy với $p=13$ thì $x^3+x+13=y^2$ có nghiệm là $(x ; y)=(4 ; 9)$. Ta sẽ chứng minh rằng phương trình $x^3+x+5=y^2$ không có nghiệm nguyên. Xét modulo 4. Có các khả năng sau xảy ra:

  • Khi $x$ chia 4 dư $0,1,2,3$, vế trái chia 4 lần lượt dư $1,3,3,3$.
  • Khi $y$ chia 4 dư $0,1,2,3$, vế phải chia 4 lần lượt dư $0,1,0,1$.

Do đó $y$ phải lẻ và $4 \mid x$. Viết biểu thức đã cho thành:

$\quad\quad\quad\quad\quad\quad\quad\quad\quad\quad (x+3)\left(x^2-3 x+10\right)=y^2+5^2$

Do $x+3 \equiv 3(\bmod 4)$ nên $x+3$ có ước nguyên tố $q \equiv 3(\bmod 4)$. Ta biết rằng với $a, b \in \mathbb{Z}$ thì $a^2+b^2$ chia hết cho số nguyên tố $q \equiv 3(\bmod 4)$ khi và chỉ khi $q \mid a$ và $q \mid b$. Từ đó thì $q \mid 5$ hay $q=5$, mâu thuẫn.

Vậy $p=13$ là số nguyên tố nhỏ nhất cần tìm.

(b) Trước hết, ta giới thiệu kết quả sau (còn gọi là định lý 4 số):

Bổ Đề. Với các số nguyên dương $a, b, c, d$ thoả mãn $a b=c d$ thì tồn tại các số nguyên dương $x, y, z, t$ sao cho $a=x y, b=z t, c=x z, d=y t$.

Chứng minh bổ đề. Đặt $k=\operatorname{gcd}(a, c)$ và viết $a=k a_1, c=k c_1$ thì rõ ràng $\operatorname{gcd}\left(a_1, c_1\right)=1$. Thay vào đề bài, ta có

$\quad\quad\quad\quad\quad\quad\quad\quad\quad\quad k a_1 b=k c_1 d \text { hay } a_1 b=c_1 d .$

Từ đây chú ý $a_1 \mid c_1 d$, nên $a_1 \mid d$, đặt $d=a_1 \ell$. Thay vào thì có $b=\ell c_1$. Từ đó, ta chọn $x=k, y=a_1, z=c_1, t=\ell$ thì có ngay điều phải chứng minh.

Quay lại bài toán, do $p$ là số chính phương nên đặt $p=a^2, a \in \mathbb{Z}$. Ta viết lại phương trình thành dạng:

$\quad\quad\quad\quad\quad\quad\quad\quad x^3+x+a^2=y^2 \text { hay } x\left(x^2+1\right)=(y-a)(y+a) .$

Áp dụng kết quả trên vào bài toán, ta thấy tồn tại các số nguyên dương $m, n, p, q$ để $x=m n, x^2+1=p q, y+a=m p, y-a=n q$. Từ đó:

$\quad\quad\quad\quad\quad\quad\quad\quad\quad\quad (m n)^2+1=p q \text { và } m p-n q=2 a \text {. }$

Xét dãy số $\left(u_n\right)$ xác định bởi $u_0=0, u_1=1, u_{n+2}=\alpha u_{n+1}+u_n$, trong đó $\alpha$ là hằng số mà ta sẽ chọn sau. Rõ ràng với mọi $n$ thì

$\quad\quad\quad\quad\quad\quad\quad\quad u_n^2-u_{n+1} u_{n-1}=(-1)^{n-1}\left(u_1^2-u_2 u_0\right)=(-1)^{n-1} .$

Khi đó, với $n$ chẵn thì $u_n^2-u_{n+1} u_{n-1}=-1$. Chọn $m n=u_{2 k}$. Ta có:

$\quad\quad\quad\quad\quad\quad\quad u_2=\alpha, u_3=\alpha^2+1, u_4=\alpha\left(\alpha^2+2\right), u_5=\alpha^4+3 \alpha^2+1$

Chọn $p=u_3, q=u_5, m n=u_4$ thì rõ ràng $(m n)^2+1=p q$. Bây giờ ta chỉ cần có được

$\quad\quad\quad\quad\quad\quad m u_3-n u_5=2 a \text { hay } m\left(\alpha^2+1\right)-n\left(\alpha^4+3 \alpha^2+1\right)=2 a .$

Từ đây chọn $\alpha=4 a^2$ và viết $m=2 a\left(\alpha^2+2\right), n=2 a\left(\alpha^4+3 \alpha^2+1\right)$ thì đẳng thức trên sẽ thoả mãn, vì

$\quad\quad\quad\quad\quad\quad\quad\quad \left(\alpha^2+1\right)\left(\alpha^2+2\right)-\left(\alpha^4+3 \alpha^2+1\right)=1 .$

Vậy phương trình có một cặp nghiệm cụ thể là

$\quad\quad (x, y)=\left(4 a^2\left(16 a^4+2\right), 2 a\left(16 a^4+2\right)\left(16 a^4+1\right)-a\right) \text { với } a=\sqrt{p} \in \mathbb{Z}^{+} .$

Bài 4. Cho tam giác $A B C$ nhọn nội tiếp đường tròn $(O)$ với $B, C$ cố định và $A$ di động trên $(O)$. $D$ là trung điểm $B C$. Trên $A B$ lấy các điểm $M, P$ và trên $A C$ lấy các điểm $N, Q$ sao cho $D A=D P=D Q$, dồng thời $D M \perp A C, D N \perp A B$.

(a) Chứng minh rằng các điểm $M, N, P, Q$ cùng thuộc một đường tròn $(\mathcal{C})$ và (C) luôn đi qua một điểm cố định.

(b) Chứng minh rằng tâm của $(\mathcal{C})$ luôn thuộc một đường tròn cố định.

Lời giải . (a) Dễ thấy tam giác $A M Q$ cân tại $M$ nên

$\quad\quad \angle D M Q=\angle D M A=90^{\circ}-\angle A=\frac{180^{\circ}-2 \angle A}{2}=\frac{180^{\circ}-\angle P D Q}{2}=\angle D P Q$

Do đó tứ giác $M P D Q$ nội tiếp. Chứng minh tương tự, ta có tứ giác $Q N D P$ nội tiếp nên $M, N, P, Q$ cùng thuộc một đường tròn $(\mathcal{C})$, và $(\mathcal{C})$ luôn đi qua điểm $D$ cố định.

(b) Gọi $K B, K C$ là hai tiếp tuyến của $(O)$. Ta có $D, K, O$ thẳng hàng, lại có:

$\quad\quad\quad\quad\quad \angle B K O=90^{\circ}-\angle B O K=90^{\circ}-\angle B A C=\angle B M D$

Từ đó tứ giác $B D K M$ nội tiếp. Để ý rằng $K D \perp B C$ nên $K M \perp A B$, hơn nữa $D N \perp A B$ nên $K M | D N$. Tương tự thì $K N | D M$. Do đó $D M K N$ là hình bình hành hay $D K, M N$ có $J$ là trung điểm chung.

Gọi $I$ là tâm của $(\mathcal{C})$ thì $I J \perp M N$ và $J L | A D$. Chú ý rằng $D$ là tâm $(A P Q)$ và cũng là trực tâm tam giác $A M N$ nên $P Q, M N$ là hai đường đối song. Đồng thời nếu $L$ là trung điểm $A D$ thì $J L$ vuông góc với đường nối hai chân đường cao từ $M, N$ của tam giác $A M N$ nên $J L \perp P Q$. Lại có $D P=D Q$ và $I P=I Q$ nên $I D \perp P Q$, do đó $J L | D I$.

Từ đây $I D L J$ là hình bình hành và $I L, D J$ có $T$ là trung điểm chung cố định. Xét phép vị tự tâm $D$ tỉ số $\frac{1}{2}$ hợp với phép đối xứng tâm $T$ thì $A \mapsto I$. Do $A$ thuộc đường tròn $(O)$ cố định nên $I$ cũng thuộc đường tròn cố định là ảnh của $(O)$ qua hợp các phép biến hình trên. Bài toán kết thúc.

Nhận xét. Bài toán này còn một hướng tiếp cận bản chất hơn như sau. Nếu gọi $A^{\prime}$ là điểm đối xứng của $A$ qua $D$ thì $K, A^{\prime}$ là hai điểm liên hợp đẳng giác trong tam giác $A B C$, từ đó đường tròn $(\mathcal{C})$ chính là đường tròn đi qua các hình chiếu của $K, A^{\prime}$ trên các cạnh tam giác $A B C$, dồng thời $I$ là trung diểm $K A^{\prime}$.

Dưới đây là một bài toán tương tự: Cho tam giác nhọn $A B C$ nội tiếp đường tròn $(O)$ có $B C$ cố định và $A$ di dộng trên $(O)$. Gọi $H$ là trực tâm tam giác và lấy điểm $E, F$ thuộc $A B, A C$ theo thứ tự đó sao cho $H$ là trung điểm $E F$.

  1. Chứng minh rằng tâm của đường tròn $(A E F)$ luôn thuộc một đường tròn cố định. Đặt là $\omega$.
  2. Giả sử $\omega$ cắt lại $(O)$ tại các điểm $X, Y$. Chứng minh rằng $X, Y, O$ thẳng hàng.

 

Ngày thi thứ hai

Bài 5. Cho số thực $a \neq 0$. Dãy số $\left(u_n\right)$ thoả mãn:

$\quad\quad\quad\quad\quad\quad\quad\quad\quad\quad u_1=0, u_{n+1}\left(u_n+a\right)=a+1 \forall n \in \mathbb{N}^*$

Tìm giới hạn của dãy số $\left(u_n\right)$.

Lời giải: Đặt $x_{n+1}=(a+1) y_n$ và $y_{n+1}=x_n+a y_n$. Ta có:

$\quad\quad\quad\quad\quad\quad\quad y_{n+2}=x_{n+1}+a y_{n+1}=a y_{n+1}+(a+1) y_n$

Đồng thời $u_n=\frac{x_n}{y_n}$. Để ý rằng $u_1=0, u_2=\frac{a+1}{a}$. Chọn $y_1=1, y_2=a$. Từ đó:

$\quad\quad\quad\quad\quad\quad\quad\quad\quad\quad y_n=\frac{(a+1)^n-(-1)^n}{a+2} \forall n \geq 1$

Công thức trên chỉ xác định với $a \neq-2$ nên xét trường hợp $a=-2$, ta có dãy

$\quad\quad\quad\quad\quad\quad\quad\quad\quad\quad \left\{\begin{array}{l}u_1=0, \\ u_{n+1}=\frac{1}{2-u_n}, n \geq 1\end{array} .\right.$

Bằng quy nạp, ta chứng minh được $u_n \in[0 ; 1)$ nên:

$\quad\quad\quad\quad\quad\quad\quad u_{n+1}-u_n=\frac{1}{2-u_n}-u_n=\frac{\left(u_n-1\right)^2}{2-u_n}>0$

Dãy $\left(u_n\right)$ tăng và bị chặn trên bởi 1 nên có giới hạn hữu hạn là $L \in(0,1)$. Giải phương trình giới hạn, ta có được $L=\frac{1}{2-L}$. Khi đó thì $L=1$.

Tiếp theo, xét $a \neq-2$, ta có:

$\quad\quad\quad\quad u_n=\frac{x_n}{y_n}=\frac{(a+1) y_{n-1}}{y_n}=\frac{(a+1)^n+(a+1)(-1)^n}{(a+1)^n-(-1)^n} \forall n \in \mathbb{N}^*$

Đặt $-(a+1)=b \in{-1 ; 1}$, ta viết lại thành:

$\quad\quad\quad\quad\quad\quad\quad\quad\quad\quad u_n=\frac{b^n-b}{b^n-1} \forall n \geq 1$

Có các khả năng sau xảy ra:

  • Nếu $b>1$ hoặc $b<-1$, tương ứng là $a<-2$ hoặc $a>0$, thì $\lim u_n=1$.
  • Nếu $-1<b<1$, tương ứng là $-2<a<0$, thì $\lim u_n=b=-(a+1)$.

Vậy ta có kết luận sau trong các trường hợp của $a$ :

  • Nếu $a \in(-2 ; 0)$ thì $\lim u_n=-(a+1)$.
  • Nếu $a \notin(-2 ; 0)$ thì $\lim u_n=-1$.

Bài 6. Tìm tất cả các hàm số $f: \mathbb{R}^{+} \rightarrow \mathbb{R}^{+}$thoả mãn diều kiện:

$\quad\quad\quad\quad\quad\quad f\left(x f\left(y^2\right)-y f\left(x^2\right)\right)=(y-x) f(x y) \forall x, y \in \mathbb{R}^{+}, x<y .$

Lời giải . Theo giả thiết thì với mọi $y>x>0$, ta đều có

$\quad\quad\quad\quad\quad\quad\quad\quad x f\left(y^2\right)-y f\left(x^2\right)>0 \Rightarrow \frac{f\left(y^2\right)}{f\left(x^2\right)}>\frac{y}{x}>1 .$

Do đó,

$\quad\quad\quad\quad\quad\quad\quad\quad y^2>x^2 \Leftrightarrow y>x \Leftrightarrow f\left(y^2\right)>f\left(x^2\right)$

nên hàm $f$ dã cho đồng biến trên $\mathbb{R}^{+}$. Trong đề bài, thay $y=x+1$, ta có

$\quad\quad\quad\quad\quad\quad\quad f\left(x f\left((x+1)^2\right)-(x+1) f\left(x^2\right)\right)=f(x(x+1))$

hay

$\quad\quad\quad\quad\quad\quad\quad\quad x f\left((x+1)^2\right)-(x+1) f\left(x^2\right)=x(x+1) $

$\quad\quad\quad\quad\quad\quad\quad\quad \Leftrightarrow \frac{f\left((x+1)^2\right)}{x+1}=\frac{f\left(x^2\right)}{x}+1, \forall x>0$

Thực hiện thao tác này nhiều lần, ta có

$\quad\quad\quad\quad\quad\quad\quad\quad \frac{f\left((x+n)^2\right)}{x+n}=\frac{f\left(x^2\right)}{x}+n, \forall x>0, n \in \mathbb{Z}^{+}$

hay

$\quad\quad\quad\quad\quad\quad\quad\quad x f\left((x+n)^2\right)-(x+n) f\left(x^2\right)=n x(x+n) .$

Trong dề bài, thay $y=x+n$, ta có

$\quad\quad\quad\quad\quad\quad f\left(x f\left((x+n)^2\right)-(x+n) f\left(x^2\right)\right)=n f(x(x+n)) $

$\quad\quad\quad\quad\quad\quad \Leftrightarrow f(n x(x+n))=n f(x(x+n)) .$

Với mọi $n \in \mathbb{Z}^{+}, y>0$, ta luôn chọn được $x>0$ để $x(x+n)=y$ nên ta có

$\quad\quad\quad\quad\quad\quad\quad\quad\quad\quad f(n y)=n f(y), \forall n \in \mathbb{Z}^{+}, y \in \mathbb{R}^{+} .$

Đặt $f(1)=a>0$, với mọi $n \in \mathbb{Z}^{+}$, cho $y=\frac{1}{n}$, suy ra

$\quad\quad\quad\quad\quad\quad\quad\quad\quad\quad f(1)=n f\left(\frac{1}{n}\right) \Rightarrow f\left(\frac{1}{n}\right)=\frac{a}{n} .$

Do đó,

$\quad\quad\quad\quad\quad\quad\quad\quad f\left(\frac{n}{m}\right)=n f\left(\frac{1}{m}\right)=\frac{n}{m} a, \forall m, n \in \mathbb{Z}^{+}$

hay $f(x)=a x, \forall x \in \mathbb{Q}^{+}$. Với mọi số thực $x_0>0$, chọn hai dãy số hữu tỷ $\left(a_n\right),\left(b_n\right)$ sao cho $a_n<x_0<b_n$ và $\lim a_n=\lim b_n=x_0$. Rõ ràng

$\quad\quad\quad\quad\quad\quad f\left(a_n\right)<f\left(x_0\right)<f\left(b_n\right) \Rightarrow a \cdot a_n<f\left(x_0\right)<a \cdot b_n,$

nên cho $n \rightarrow+\infty$, ta có $f\left(x_0\right)=a x_0$. Do đó, với mọi số thực $x>0$ thì $f(x)=a x$. Thay vào biểu thức đã cho, ta có

$\quad\quad\quad\quad\quad \left\{\begin{array}{l}f\left(x f\left(y^2\right)-y f\left(x^2\right)\right)=a^2\left(x y^2-x^2 y\right)=a^2(y-x) x y \\ (y-x) f(x y)=a(y-x) x y\end{array}\right.$

nên $a=1$. Vậy tất cả các hàm số cần tìm là $f(x)=x, \forall x>0$.

Nhận xét. Có một điều đáng chú ý ở bài toán này là việc từ giả thiết, ta phải ngầm hiểu rằng $x f\left(y^2\right)-y f\left(x^2\right)>0$ với mọi cặp số dương $x<y$. Ta có thể thêm tường minh điều kiện đó vào đề bài cho rõ. Tuy nhiên, nếu thêm theo kiểu như sau thì sẽ có một chút vấn đề phát sinh:

Tìm tất cả các hàm số $f: \mathbb{R}^{+} \rightarrow \mathbb{R}^{+}$thoả mãn với mọi cặp số dương $x<y$, nếu $x f\left(y^2\right)-y f\left(x^2\right)>0$ thì

$\quad\quad\quad\quad\quad\quad\quad\quad\quad\quad f()=(y-x) f(x y) \forall x, y \in \mathbb{R}^{+}, x<y .$

Khi đó, ta có thể nhận thêm một hàm số thỏa mãn nữa là $f(x)=\sqrt{x}$. Lý do là vì với mọi cặp số $y>x>0$, ta đều có $x f\left(y^2\right)-y f\left(x^2\right)=0$, mà vì thế, điều kiện “nếu” ở trên là sai nên mệnh đề kéo theo là đúng.

Bài 7. Cho $n=2018.2019$. Gọi $A$ là tập hợp các bộ $\left(a_1, a_2, \ldots, a_n\right)$ có thứ tự thoả mãn điều kiện $a_i \in{0,1} \forall i \in{1,2, \ldots, n}$ và $\sum_{i=1}^n a_i=2018^2$.

Có bao nhiêu bộ $\left(a_1, a_2, \ldots, a_n\right)$ từ $A$ dể:

$\quad\quad\quad\quad\quad \sum_{i=1}^k a_i \geq \frac{a}{2} \text { và } \sum_{i=n-k+1}^n a_i \geq \frac{k}{2} \forall k \in{1,2, \ldots, n}$

Lời giải. Ta giải bài toán tổng quát khi thay 2018 bởi $m \in \mathbb{Z}^{+}$. Bài toán đã cho tương đương với bài toán sau:

Trong hệ trục tọa độ Oxy, xét lưới điểm nguyên trong hình chũ nhật có đỉnh dưới bên trái là $O(0 ; 0)$ và dỉnh trên bên phải là $A\left(m^2 ; m\right)$. Dặt $B(m ; m)$ và $C\left(m^2-m ; 0\right)$, hỏi có bao nhiêu đương đi tù̀ $O \rightarrow A$ sao cho mỗi bước, ta đi sang phải hoặc lên trên 1 đơn vị, gọi là đương đi đơn, và không vượt lên trên $O B$ cũng nhu không xuống dưới $A C$ ?

Ở đây, các số $0 ; 1$ tương ứng với các bước đi lên trên, các bước đi sang phải; còn điều kiện tổng $k$ số đầu và tổng $k$ số cuối không nhỏ hơn $\frac{k}{2}$ tương ứng với số lượng bước đi lên không vượt quá số lượng bước đi sang phải. Để thuận tiện, ta gọi đường đi cắt $d$ nếu nó có các phần nằm về cả hai phía của $d$. Trước hết, ta sẽ chứng minh bổ đề sau:

Bổ Đề. Số đường đi đơn từ $O \rightarrow A(m ; n)$, có cắt đường thẳng $y=x$, là $C_{m+n}^{m+1}$.

Thật vậy, Xét đường thẳng $(d): y=x+1$, rõ ràng các đường đi đơn cắt $y=x$ dều sẽ có điểm chung với đường thẳng $(d)$ này. Tại các điểm chung đó, ta thực hiện đối xứng trục để được một đường đi mới xuất phát từ $O \rightarrow A^{\prime}(n-1, m+1)$.

Trong hình trên, đường cũ là đứt nét, còn đường mới là liền nét. Rõ ràng phép đối xứng trục trên là song ánh, biến các đường cần tìm (cắt $y=x$ ), thành các đường từ $O \rightarrow A^{\prime}$; do đó, số lượng đường cần tìm là $C_{m+n}^{n-1}$.

Trở lại bài toán,

Số đường đi đơn từ $O \rightarrow A\left(m^2 ; m\right)$ là $C_{m^2+m}^m$ vì nó bằng số cách chọn $m$ lần đi lên trong tổng số $m^2+m$ lần di chuyển, trong đó số đường đi cắt $O B$ bằng số đường đi cắt $A C$ và bằng $C_{m^2+m}^{m-1}$ (theo bổ đề).

Do đó, ta chỉ cần tìm số đường đi cắt cả $O B, A C$ với ý tưởng đối xứng hai lần đã dùng để chứng minh bổ đề.

Đầu tiên, ta thực hiện đối xứng qua đường thẳng $y=x+1$; khi đó, các đường đi đơn sẽ xuất phát từ $O \rightarrow A^{\prime}\left(m-1 ; m^2+1\right)$. Do các đường ban đầu còn vượt qua $A C$ nên các đường mới phải cắt thêm $y=x+m^2-m+3$. Tiếp tục đối xứng qua đường thẳng này, ta đưa về đếm số đường đi đơn từ $O \rightarrow A^{\prime \prime}\left(m-2, m^2+2\right)$. Suy ra số đường đi trong trường hợp này là $C_{m^2+m}^{m-2}$. Vậy theo nguyên lý bù trừ, kết quả cần tìm sẽ là

$\quad\quad\quad\quad\quad\quad\quad\quad\quad\quad C_{m^2+m}^m-2 C_{m^2+m}^{m-1}+C_{m^2+m}^{m-2} .$

Thay $m=2018$, ta có số lượng đường đi, cũng chính là số bộ thỏa mãn đề bài.

Nhận xét. Dưới đây là một số kết quả tương tự về đường đi đơn trong đề bài

$1$. Số đường đi đơn từ $(0 ; 0) \rightarrow(m ; n)$ mà không có điểm chung với $y=x$ là

$\quad\quad\quad\quad\quad\quad\quad\quad\quad\quad\quad\quad \frac{m-n}{m+n} C_{m+n}^m .$

$2$. Số đường đi đơn từ $(0 ; 0) \rightarrow(m ; n)$ mà không vượt qua $y=x$ là

$\quad\quad\quad\quad\quad\quad\quad\quad\quad\quad\quad C_{m+n}^n-C_{m+n}^{n-1} .$

$3$. Số đường đi gồm $n$ bước mà không vượt $y=x$ là

$\quad\quad\quad\quad\quad\quad\quad\quad\quad \sum_{i=n / 2}^n \frac{n !(2 i+1-n)}{(i+1) !(n-i) !}=C_n^{[n / 2]} .$

$4$. Số đường đi đơn từ $(0 ; 0) \rightarrow(m ; n)$ mà không có điểm chung với $y=x+t$ là

$\quad\quad\quad\quad\quad\quad\quad\quad\quad\quad\quad\quad C_{m+n}^n-C_{m+n}^{m-t}$.

Bạn đọc có thể dùng phương pháp tương tự trên để giải quyết các bài toán này.

Bài 8. Đường tròn $(\mathcal{C})$ tâm $I$ nội tiếp tam giác $A B C$ và tiếp xúc với các cạnh $A B, A C$ tại $E, F$. $A M, A N$ là các đường phân giác trong, phân giác ngoài của góc $\angle B A C(M, N$ nằm trên $B C)$. Gọi $d_M, d_N$ lần lượt là các tiếp tuyến của $(\mathcal{C})$ qua $M, N$ và khác $B C$.

(a) Chứng minh rằng $d_M, d_N, E F$ dồng quy tại điểm $D$.

(b) Lấy trên $A B, A C$ các điểm $P, Q$ thoả mãn $D P|A C, D Q| A B$. Gọi $R, S$ là trung điểm của $D E, D F$. Chứng minh rằng $I$ thuộc đường thẳng qua các trực tâm của hai tam giác $D P S, D Q R$.

Lời giải. (a) Gọi $X, Y$ lần lượt là tiếp điểm của tiếp tuyến thứ hai kẻ từ $M$ dến $(I)$ và $D^{\prime}$ là tiếp điểm của $(I)$ trên $B C$. Gọi $K$ là trung điểm $E F$.

Xét trong đường tròn $(I)$ thì $E F$ là đường đối cực của $A$ và $K \in E F$ nên đối cực của $K$ sẽ đi qua $A$, mà $N A \perp I A$ nên $N A$ chính là đường đối cực của $K$.

Đường đối cực của $K$ đi qua $N$ nên đối cực của $N$, là $D^{\prime} Y$, sẽ đi qua $K$. Dễ thấy rằng $A M$ là trục đối xứng của tứ giác $D^{\prime} X E F$ nên suy ra $D^{\prime} X | E F$. Xét $D^{\prime}(E F, X Y)$, ta có có $D^{\prime} Y$ đi qua trung điểm của $E F$ và $D^{\prime} X | E F$ nên

$\quad\quad\quad\quad\quad\quad\quad\quad\quad\quad D^{\prime}(E F, X Y)=-1$

hay tứ giác $E X F Y$ điều hòa. Suy ra $M X, N Y, E F$ đồng quy. Ngoài ra ta cũng có $X, Y, A$ thẳng hàng.

(b) Dễ thấy các tam giác $P E D$ và $D Q F$ là các tam giác cân. Gọi $H_1, H_2$ lần lượt là trực tâm của tam giác $\triangle D P S, \triangle D Q R$. Ta có

$\quad\quad\quad\quad\quad\quad\quad\quad \angle P H_1 S=\angle P D F=\angle A F E=\angle P E S$

nên $E P S H_1$ là tứ giác nội tiếp. Suy ra $R H_1 \cdot R P=R S \cdot R E$. Ngoài ra,

$\quad\quad\quad\quad\quad\quad\quad\quad\quad\quad K A \cdot K I=K E \cdot K F$

nên

$\quad\quad\quad\quad\quad\quad\quad\quad\quad \frac{R P}{K A} \cdot \frac{R H_1}{K I}=\frac{R E}{K E} \cdot \frac{R S}{K F} .$

Theo định lý Thales thì $\frac{R P}{K A}=\frac{R E}{K E}$ nên $\frac{R H_1}{K I}=\frac{R S}{K F}$, mà

$\quad\quad\quad\quad\quad\quad\quad R S=R D-S D=\frac{D E-D F}{2}=\frac{E F}{2}=K F$

Suy ra $R H_1=K I$, mà $R H_1 | K I$ (do cùng vuông góc với $E F$ ) nên $I K R H_1$ là hình chữ nhật, kéo theo $I H_1 | E F$. Một cách tương tự, ta có $I H_2 | E F$ vậy nên đường thẳng $H_1 H_2$ đi qua $I$.

Nhận xét. Trong câu a, tính chất $A, X, Y$ thẳng hàng của bài toán cũng đúng khi thay $M, N$ là chân các đường phân giác bởi cặp điểm liên hợp điều hòa bất kỳ với $B, C$. Điều này có được nhờ tính chất của các đường đối cực (hoặc có thể chứng minh nhờ việc sử dụng phép chiếu trực giao các chùm điều hòa).

 

 

 

 

 

 

 

 

 

 

 

 

 

 

 

 

 

 

 

 

 

 

 

 

 

 

 

 

 

 

 

 

 

 

 

 

 

 

 

 

 

 

 

 

 

 

 

 

 

 

 

 

 

 

 

 

 

 

 

 

 

 

 

 

 

 

 

 

 

 

 

 

 

 

 

 

 

 

 

 

 

 

 

 

 

 

 

 

 

 

 

 

 

 

 

 

 

 

 

 

 

 

 

 

 

 

 

 

 

 

 

 

 

 

 

 

 

 

 

 

 

 

 

 

Đề thi và đáp án kì thi chọn đội tuyển thi Quốc gia trường Phổ thông Năng khiếu năm học 2014 – 2015

ĐỀ THI

 

Ngày thi thứ nhất

Bài 1. Cho $a, b, c>0$ thỏa mãn điều kiện $(a+1)(b+1)(c+1)=1+4 a b c$. Chứng mình rằng ta có bất đẳng thức $a+b+c \leq 1+a b c$.

Bài 2. Cho tập hợp $A=[n^3-4 n+15 \mid n \in \mathbb{N}]$. Tìm tất cả các phần tử $a \in A$ thỏa mãn đồng thời hai điều kiện sau đây:

$\quad (i)\quad a$ là số chẵn.

$\quad (ii)$ Nếu $a_1, a_2$ là các ước số của $\frac{a}{2}$ với $a_1, a_2>1$ thì $\operatorname{gcd}\left(a_1, a_2\right)>1$.

Bài 3. Tìm tất cả các hàm số $f: \mathbb{N}^* \rightarrow \mathbb{N}^*$ thỏa mãn:

$\quad\quad\quad\quad\quad\quad\quad\quad\quad\quad f\left(\frac{f(n)}{n}\right)=n^2 \forall n \in \mathbb{N}^*$

Bài 4. Cho tam giác $A B C$ nội tiếp $(O)$, có $B, C$ cố định và $A$ thay đổi trên $(O)$. Ký hiệu $(I)$ là đường tròn nội tiếp tam giác $A B C$. Gọi $\left(O_1\right)$ là đường tròn qua $A, B$ và tiếp xúc với đường tròn $(I)$ tại $E$. Gọi $\left(O_2\right)$ là đường tròn qua $A, C$ và tiếp xúc với đường tròn $(I)$ tại $F$. Đường phân giác trong của góc $\angle A E B$ cắt $\left(O_1\right)$ tại $M$ và đường phân giác trong của góc $\angle A F C$ cắt $\left(O_2\right)$ tại $N$.

(a) Chứng minh rằng tứ giác $E F M N$ nội tiếp.

(b) Gọi $J$ là giao điểm của $E M$ và $F N$. Chứng minh rằng đường thẳng $I J$ luôn đi qua một điểm cố định.

Ngày thi thứ hai

Bài 5. Cho dãy số $\left(x_n\right)$ bởi $x_0=1, x_1=2014$ và $x_{n+1}=\sqrt[3]{x_n x_{n-1}^2} \forall n \in \mathbb{N}^*$.

(a) Chứng minh rằng dãy số $\left(x_n\right)$ có giới hạn hữu hạn và tìm giới hạn đó.

(a) Với mỗi $n \geq 2$, hãy tìm số nguyên dương $k$ nhỏ nhất sao cho $a=x_n^k$ là một số nguyên. Chứng minh rằng khi đó $a$ không thể viết được dưới dạng tổng các lũy thừa bậc ba của hai số tự nhiên.

Bài 6. Cho $X$ là tập hợp gồm 19 phần tử.

(a) Chứng minh rằng tồn tại ít nhất 2600 tập con 7 phần tử của $X$ sao cho với hai tập con $A, B$ bất kỳ trong số 2600 tập con đó, ta có $|A \cap B| \leq 5$.

(b) Xét một họ $\Omega$ gồm $k$ tập con có 7 phần tử của $X$. Một tập $A \subset X$ được gọi là một cận trên của $\Omega$ nếu như $|A|=8$ và tồn tại một tập con $F$ của họ $\Omega$ sao cho $F \subset A$. Gọi $d$ là số tập con cận trên của họ $\Omega$. Chứng minh rằng $d \geq \frac{3}{2} k$.

Bài 7. Cho tam giác $A B C$ không cân. Gọi $I$ là trung điểm $B C$. Đường tròn $(I)$ tâm $I$ đi qua $A$ cắt $A B, A C$ lần lượt tại $M, N$. Giả sử $M I, N I$ cắt $(I)$ tại $P, Q$. Gọi $K$ là giao điểm của $P Q$ với tiếp tuyến tại $A$ của $(I)$. Chứng minh rằng $K$ thuộc đường thẳng $B C$.

Bài 8. Tìm số nguyên dương $n$ lớn nhất thỏa mãn các điều kiện sau:

$\quad (i)\quad n$ không chia hết cho 3 .

$\quad (ii)$ Bảng vuông $n \times n$ không thể được phủ kín bằng 1 quân tetramino $1 \times 4$ và các quân trimino $1 \times 3$. Trong phép phủ, các quân tetramino và trimino được phép quay dọc nhưng không được phép chườm lên nhau hoặc nằm ra ngoài bảng vuông.

LỜI GIẢI

Bài 1. Cho $a, b, c>0$ thỏa mãn điều kiện $(a+1)(b+1)(c+1)=1+4 a b c$. Chứng minh rằng ta có bất đẳng thức
$\quad\quad\quad\quad\quad\quad\quad\quad\quad\quad a+b+c \leq 1+a b c .$
Lời giải. Điều kiện đã cho viết thành $a b+b c+c a+a+b+c=3 a b c$. Chia hai vế cho $a b c$ rồi đặt $a=\frac{1}{x}, b=\frac{1}{y}, c=\frac{1}{z}$, ta có $x y+y z+z x+x+y+z=3$.
Bất đẳng thức đã cho có thể viết thành
$\quad\quad\quad\quad\quad\quad x y+y z+z x-x y z \leq 1 \text { hay } x+y+z+x y z \geq 2 \text {. }$
Theo bất đẳng thức Schur thì
$\quad\quad\quad\quad\quad\quad (x+y+z)^3+9 x y z \geq 4(x y+y z+z x)(x+y+z) .$
Đặt $m=x+y+z, n=x y+y z+z x$ thì $m+n=3$ và
$\quad\quad\quad\quad\quad\quad\quad\quad\quad\quad x y z \geq \frac{4 m n-m^3}{9} .$
Ta sẽ chứng minh rằng
$\quad\quad\quad\quad\quad\quad m+\frac{4 m n-m^3}{9} \geq 2 \Leftrightarrow m^3+4 m^2-21 m+18 \leq 0$
hay $(m-2)\left(m^2+6 m-9\right) \leq 0$. Chú ý rằng $m^2 \geq 3 n$ nên
$\quad\quad\quad\quad\quad\quad\quad\quad m^2 \geq 3(3-m) \Leftrightarrow m^3+3 m \geq 9$

Do đó $m^2+6 m-9 \geq 0$. Ta xét các trường hợp

  1. Nếu $m>2$ thì $x+y+z>2$ nên hiển nhiên bất đẳng thức cần chứng minh là đúng.
  2. Nếu $m \leq 2$ thì $m-2 \leq 0$ nên ta cũng có $(m-2)\left(m^2+6 m-9\right) \leq 0$.

Vậy trong mọi trường hợp, ta luôn có điều phải chứng minh.

Bài 2. Cho tập hợp $A=[n^3-4 n+15 \mid n \in \mathbb{N}]$. Tìm tất cả các phần tử $a \in A$ thỏa mãn đồng thời hai điều kiện sau đây:

$\quad (i)\quad a$ là số chẵn.

$\quad (ii)$ Nếu $a_1, a_2$ là các ước số của $\frac{a}{2}$ với $a_1, a_2>1$ thì $\operatorname{gcd}\left(a_1, a_2\right)>1$.

Lời giải. Ta thấy rằng $a=n^3-4 n+15$ chẵn nên $n^3+15$ chẵn hay $n$ lẻ. Đặt $n=2 k+1$ với $k \in \mathbb{N}$. Ta có

$\quad\quad\quad\quad\quad\quad a=n^3-4 n+15 =(n+3)\left(n^3-3 n+15\right) $

$\quad\quad\quad\quad\quad\quad\quad\quad\quad\quad\quad\quad\quad\quad =(2 k+4)\left(4 k^2-2 k+3\right)$

nên $\frac{a}{2}=(k+2)\left(4 k^2-2 k+3\right)$. Điều kiện ii) cho thấy rằng $\frac{a}{2}$ phải là lũy thừa của một số nguyên tố, vì nếu nó có hai ước nguyên tố trở lên, đặt là $p, q$ thì chọn $x=p, y=q$, ta có $x, y>1$ nhưng $\operatorname{gcd}(x, y)=1$, không thỏa mãn.

Vì $\left(4 k^2-2 k+3\right)-(k+2)=4 k^2-3 k+1>0$ với mọi $k \in \mathbb{N}$. Do đó, ta phải có $k+2 \mid 4 k^2-2 k+3$. Suy ra

$\quad\quad\quad\quad\quad\quad\quad\quad\quad\quad\frac{4 k^2-2 k+3}{k+2}=4 k-10+\frac{23}{k+2} \in \mathbb{Z} .$

Do đó $k+2 \in{1,23}$ vì $k+2>0$. Ta xét các trường hợp

  1. Nếu $k+2=1$ thì $k=-1$ hay $n=2 k+1=-1<0$, không thỏa mãn.
  2. Nếu $k+2=23$ thì $k=21$ hay $n=43$, tính được $\frac{a}{2}=3 \cdot 5^2 \cdot 23^2$, cũng không thỏa mãn.

Vậy không tồn tại số $a$ nào thỏa mãn.

Bài 3. Tìm tất cả các hàm số $f: \mathbb{N}^* \rightarrow \mathbb{N}^*$ thỏa mãn:

$\quad\quad\quad\quad\quad\quad\quad\quad\quad\quad f\left(\frac{f(n)}{n}\right)=n^2 \forall n \in \mathbb{N}^* .$

Lời giải. Với $n \in \mathbb{N} *$, ta thấy rằng nếu $n=1$ thì $f(f(1))=1$.

Nếu $n>1$ thì gọi $p$ là một ước nguyên tố bất kỳ của $n$.

Vì $\frac{f(n)}{n} \in \mathbb{N} *$ nên $n \mid f(n)$. Đặt $a=v_p(n), b=v_p(f(n))$ thì trước hết, ta có $a \leq b$.

Từ $f\left(\frac{f(n)}{n}\right)=n^2$, ta suy ra rằng $\frac{f(n)}{n} \mid n^2$ hay $f(n) \mid n^3$, tức là $b \leq 3 a$.

Trong biểu thức đã cho, thay $n \rightarrow \frac{f(n)}{n}$ thì

$\quad\quad\quad\quad\quad\quad f\left(\frac{f\left(\frac{f(n)}{n}\right)}{\frac{f(n)}{n}}\right)=\left(\frac{f(n)}{n}\right)^2 \Leftrightarrow f\left(\frac{n^3}{f(n)}\right)=\left(\frac{f(n)}{n}\right)^2$

Do đó, ta phải có

$\quad\quad\quad\quad\quad\quad\quad\quad\quad\left(\frac{f(n)}{n}\right)^2\left|\frac{n^3}{f(n)} \Leftrightarrow f^3(n)\right| n^5 \text { nên } 3 b \leq 5 a \text {. }$

Sau đó lại tiếp tục thay $n$ trong biểu thức đã cho bởi $\frac{n^5}{f^3(n)}$ và cứ như thế, ta xây dựng được hai dãy hệ số của $a, b$ như sau

$\quad\quad\quad\quad\quad\quad\quad\quad\quad\quad u_0=v_0=1, u_1=3, v_1=1 \text { và } $

$\quad\quad\quad\quad\quad\quad\quad\quad u_{k+1}=2 u_{k-1}+v_k, v_{k+1}=2 v_{k-1}+u_k \text { với } k \geq 1 .$

Khi đó

$\quad\quad\quad\quad\quad\quad\quad\quad\quad\quad\quad\quad\frac{v_{2 k}}{u_{2 k}} \leq \frac{b}{a} \leq \frac{u_{2 k+1}}{v_{2 k+1}} .$

Biến đổi công thức của hai dãy, ta có $u_{n+2}=5 u_n-4 u_{n-2}, v_{n+2}=5 v_n-4 v_{n-2}$ và cả hai dãy đều có phương trình đạạc trưng là $t^2-5 t+4=0$. Ngoài ra, dãy chăn và dãy lẻ trong mỗi dãy đều độc lập với nhau.

Ta có $u_0=1, u_2=3, v_0=1, v_2=5$ nên

$\quad\quad\quad\quad\quad\quad\quad\quad u_{2 k}=\frac{13+2 \cdot 16^k}{15}, v_{2 k}=\frac{11+4 \cdot 16^k}{15}, k \geq 1 .$

Từ đó, dễ dàng tính được $\lim \frac{u_{2 k+1}}{v_{2 k+1}}=2$.

Một cách tương tự, ta tính được $\lim \frac{u_{2 k}}{v_{2 k}}=\frac{1}{2}$. Do đó, số $\frac{b}{a}$ bị kẹp ở giữa và là số nguyên nên chỉ có thể là $\frac{b}{a}=2 \Leftrightarrow b=2 a$.

Rõ ràng tập hợp ước nguyên tố của $n$ và $f(n)$ là giống nhau. Hơn nữa, với một ước nguyên tố cụ thể thì số mũ trong $f(n)$ gấp đôi số mũ trong $n$. Suy ra $f(n)=$ $n^2, \forall n>1$.

Tiếp theo, giả sử $f(1)=n>1$ thì ta có $f(f(1))=1$ nên $f(n)=1$, mâu thuẫn. Vì thế nên chỉ có thể $f(1)=1$.

Vậy tất cả các hàm thỏa mãn là $f(n)=n^2, \forall n \in \mathbb{N}^*$.

Bài 4. Cho tam giác $A B C$ nội tiếp $(O)$, có $B, C$ cố định và $A$ thay đổi trên $(O)$. Ký hiệu $(I)$ là đường tròn nội tiếp tam giác $A B C$. Gọi $\left(O_1\right)$ là đường tròn qua $A, B$ và tiếp xúc với đường tròn $(I)$ tại $E$. Gọi $\left(O_2\right)$ là đường tròn qua $A, C$ và tiếp xúc với đường tròn $(I)$ tại $F$. Đường phân giác trong của góc $\angle A E B$ cắt $\left(O_1\right)$ tại $M$ và đường phân giác trong của góc $\angle A F C$ cắt $\left(O_2\right)$ tại $N$.

(a) Chứng minh rằng tứ giác $E F M N$ nội tiếp.

(b) Gọi $J$ là giao điểm của $E M$ và $F N$. Chứng minh rằng đường thẳng $I J$ luôn đi qua một điểm cố định.

Lời giải. (a) Trước hết, ta thấy rằng $O_1, I, E$ thẳng hàng và $O_2, I, F$ thẳng hàng. Vì $M$ là trung điểm cung $A B$ của $\left(O_1\right)$ nên $O_1 M$ là trung trực của $A B$, suy ra $O \in O_1 M$. Tương tự, ta cũng có $O \in O_1 N$.

Gọi $P, Q$ lần lượt là tiếp điểm của $(I)$ với $A B, A C$.

Vì $I P | O_1 M$ (cùng vuông góc với $A B$ ) nên $\angle M O_1 E=\angle P I E$. Hơn nữa, các tam giác $O_1 M E, I P E$ đều cân với đỉnh là $O_1, I$ nên suy ra chúng đồng dạng, tức là $\angle I E P=\angle O_1 E M$ hay $E, P, M$ thẳng hàng. Tương tự thì $F, Q, N$ cũng thẳng hàng.

Vì ta đã có $E, F, P, Q$ cùng thuộc đường tròn $(I)$ nên để có $E, F, M, N$ cùng thuộc một đường tròn thì $\angle E M N=\angle E F N=\angle E P Q$ hay $M N | P Q$.

Mặt khác, $A I \perp P Q$ nên ta cần có $A I \perp M N$.

Thật vậy, sử dụng phương tích với đường tròn $(I)$ ta có

$\quad\quad\quad\quad\quad\quad M A^2-N A^2=M P \cdot M E-N Q \cdot N F=M I^2-N I^2$

nên theo định lý bốn điểm thì $A I \perp M N$, từ đó ta có điều phải chứng minh.

(b) Vì $P Q|M N, O M| I P$ nên dễ dàng có $\angle I P Q=\angle O M N$. Tương tự $\angle I P Q=$ $\angle O N M$.

Do đó, hai tam giác $I P Q, O M N$ đồng dạng với nhau, tức là

$\quad\quad\quad\quad\quad\quad\quad\quad\quad\quad\quad\quad\frac{I P}{O M}=\frac{P Q}{M N} \text {. }$

Ngoài ra,

$\quad\quad\quad\quad\quad\quad\quad\quad\quad\quad\quad\quad\frac{J P}{J M}=\frac{I P}{O M}$

kết hợp với $\angle J P I=\angle J M O$, ta có hai tam giác $J P I, J M O$ đồng dạng, dẫn đến

$\quad\quad\quad\quad\quad\quad\quad\quad\quad\quad\quad\quad\angle P J I=\angle M J O \text {. }$

Từ đây suy ra $I, J, O$ thẳng hàng hay $I J$ luôn đi qua điểm $O$ cố định.

Ngày thi thứ hai

Bài 5. Cho dãy số $\left(x_n\right)$ bởi $x_0=1, x_1=2014$ và $x_{n+1}=\sqrt[3]{x_n x_{n-1}^2} \forall n \in \mathbb{N}^*$.

(a) Chứng minh rằng dãy số $\left(x_n\right)$ có giới hạn hữu hạn và tìm giới hạn đó.

(b) Với mỗi $n \geq 2$, hãy tìm số nguyên dương $k$ nhỏ nhất sao cho $a=x_n^k$ là một số nguyên. Chứng minh rằng khi đó $a$ không thể viết được dưới dạng tổng các lũy thừa bậc ba của hai số tự nhiên.

Lời giải. (a) Đặt $u_n=\log _{2014}\left(x_n\right)$ thì ta thu được dãy $\left(u_n\right)$ như sau

$\quad\quad\quad\quad\quad\quad\quad\quad\quad\quad\left\{\begin{array}{c}u_0=0, u_1=1 \\ u_{n+1}=\frac{1}{3} u_n+\frac{2}{3} u_{n-1}\end{array}\right.$

Từ đó tìm được

$\quad\quad\quad\quad\quad\quad\quad\quad\quad\quad u_n=\frac{3}{5}-\frac{3}{5} \cdot\left(\frac{-2}{3}\right)^n$

Suy ra $\lim _{n \rightarrow+\infty} u_n=\frac{3}{5}$ nên ta có được

$\quad\quad\quad\quad\quad\quad\quad\quad \lim _{n \rightarrow+\infty} x_n=\lim _{n \rightarrow+\infty}\left(2014^{u_n}\right)=2014^{3 / 5}$

(b) Ta thấy rằng để có $\left(x_n\right)^k$ là một số nguyên thì $\frac{3 k\left(3^n-(-2)^n\right)}{5 \cdot 3^n} \in \mathbb{Z}$ nguyên. Ta xét các trường hợp

  1. Nếu $n$ lẻ thì $3^n-(-2)^n=3^n+2^n: 5$. Vì $\operatorname{gcd}\left(\frac{3^n+2^n}{5}, 3^n\right)=1$ nên ta được $3^n \mid 3 k$ nên $k$ nhỏ nhất thỏa mãn điều này là $k=3^{n-1}$.
  2. Nếu $n$ chẵn thì $3^n-2^n \equiv(-2)^n-2^n=0(\bmod 5)$ và tương tự, ta cũng tìm được $k=3^{n-1}$.

Do đó số $k$ nhỏ nhất cần tìm là $k=3^{n-1}$. Tiếp theo, ta sẽ chứng minh rằng phương trình sau không có nghiệm tự nhiên

$\quad\quad\quad\quad\quad\quad\quad a^3+b^3=2014^n \Leftrightarrow(a+b)\left(a^2-a b+b^2\right)=2014^n$

Gọi $n_0$ số nguyên dương nhỏ nhất sao cho tồn tại $a, b \in \mathbb{Z}^{+}$để $a^3+b^3=2014^{n_0}$. Dễ thấy $n_0=1$ không thỏa mãn nên ta chỉ xét $n_0 \geq 2$. Ta xét các trường hợp

  1. Nếu $\operatorname{gcd}\left(a+b, a^2-a b+b^2\right)=1$ thì dễ thấy $(a-b)^2 \geq 1$. Khi đó

$\quad\quad\quad\quad\quad\quad\quad\quad\quad a^2-a b+b^2 \geq a+b>\sqrt{a^2-a b+b^2} .$

Vì $2014=2 \cdot 19 \cdot 53$ nên chỉ có thể xảy ra

$\quad\quad\quad\quad\quad\quad\quad\quad\quad\quad a+b=19^{n_0}, a^2-a b+b^2=106^{n_0} .$

Ngoài ra $(a+b)^2 \leq 4\left(a^2-a b+b^2\right)$ nên ta phải có $361^{n_0} \leq 4 \cdot 106^{n_0}$. Đánh giá này sai khi $n_0 \geq 2$ nên trường hợp này không thỏa mãn.

$2$. Nếu $\operatorname{gcd}\left(a+b, a^2-a b+b^2\right)>1$ thì chẳng hạn $a+b=2^x u, a^2-a b+b^2=2^y v$ với $\quad\quad\quad\quad\quad\quad\quad\quad\operatorname{gcd}(u, 2)=\operatorname{gcd}(v, 2)=1 .$

Các trường hợp còn lại chứng minh tương tự. Ngoài ra

$\quad\quad\quad\quad\quad\quad\quad\quad\quad\quad u v=1007^{n_0}, x+y=n_0 .$

Chú ý rằng $(a+b)^2-\left(a^2-a b+b^2\right)=3 a b$ nên $3 a b$ cũng chẵn, tức là cả hai số $a, b$ đều chẵn (vì nếu không thì $a^3+b^3$ lẻ).

Từ đây dễ dàng chứng minh được $3 v_2(a)=3 v_2(b)=n_0$, ta đưa về $x^{\prime 3}+y^{\prime 3}=$ $1007^{n_0}$. Cứ như thế, ta được 2014 $|a, 2014| b$ nên phương trình sau cũng có nghiệm nguyên dương

$\quad\quad\quad\quad\quad\quad\quad\quad\quad\quad \left(\frac{a}{2014}\right)^3+\left(\frac{b}{2014}\right)^3=2014^{n_0-3} .$

Điều này mâu thuẫn với các chọn $n_0$ nên phương trình trên vô nghiệm. Các trường hợp còn lại tương tự.

Ta có điều phải chứng minh.

Bài 6. Cho $X$ là tập hợp gồm 19 phần tử.

(a) Chứng minh rằng tồn tại ít nhất 2600 tập con 7 phần tử của $X$ sao cho với hai tập con $A, B$ bất kỳ trong số 2600 tập con đó, ta có $|A \cap B| \leq 5$.

(b) Xét một họ $\Omega$ gồm $k$ tập con có 7 phần tử của $X$. Một tập $A \subset X$ được gọi là một cận trên của $\Omega$ nếu như $|A|=8$ và tồn tại một tập con $F$ của họ $\Omega$ sao cho $F \subset A$. Gọi $d$ là số tập con cận trên của họ $\Omega$. Chứng minh rằng $d \geq \frac{3}{2} k$.

Lời giải. (a) Không mất tính tổng quát, ta có thể giả sử $X$ là tập hợp 19 số nguyên dương đầu tiên. Gọi $X(k)$ là tập hợp tất cả các tập con có 7 phần tử của $X$ và tổng các phần tử của nó chia 19 dư $k$.

Khi đó, dễ thấy rằng $|X(0)|+|X(1)|+\cdots+|X(18)|$ chính là số tập con có 7 phần tử tùy ý của $X$ và là $C_{19}^7$.

Ta thấy rằng hai tập hợp $A, B \in X(k)$ tùy ý đều thỏa mãn đề bài.

Thật vậy,

Giả sử $|A \cap B|=6$ (không thể có $|A \cap B|=7$ vì khi đó hai tập hợp trùng nhau).

Đặt $A=[a_1, a_2, a_3, a_4, a_5, a_6, x]$, $B=[a_1, a_2, a_3, a_4, a_5, a_6, y]$ thì

$\quad\quad\quad\quad\quad\quad\quad\quad\quad\quad \sum_{i=1}^6 a_i+x \equiv \sum_{i=1}^6 a_i+y \equiv k \quad(\bmod 19)$

nên $x \equiv y(\bmod 19)$. Suy ra $x=y$, mâu thuẫn. Đến đây, dễ thấy rằng

Ta có điều phải chứng minh.

(b) Xét một tập hợp $F$ thuộc họ $\Omega$. Vì $|X \backslash F|=19-7=12$ nên có tất cả 12 tập hợp $A \subset X$ với $|A|=8$ và $F \subset A$.

Ngược lại, ứng với một tập hợp $A$ là một cận trên của họ $\Omega$, có không quá 8 tập $F$ trong họ $\Omega$ sao cho $F \subset A$. Do đó $d \geq \frac{12}{8} k$ hay $d \geq \frac{3}{2} k$.

Đẳng thức xảy ra khi họ $\Omega$ là tập hợp tất cả các tập con có 7 phần tử của $X$.

Bài 7. Cho tam giác $A B C$ không cân. Gọi $I$ là trung điểm $B C$. Đường tròn (I) tâm $I$ đi qua $A$ cắt $A B, A C$ lần lượt tại $M, N$. Giả sử $M I, N I$ cắt $(I)$ tại $P, Q$. Gọi $K$ là giao điểm của $P Q$ với tiếp tuyến tại $A$ của $(I)$. Chứng minh rằng $K$ thuộc đường thẳng $B C$.

Lời giải. Không mất tính tổng quát, giả sử $A B<A C$.

Kẻ đường kính $A J$ của đường tròn $(I)$. Khi đó, dễ thấy tứ giác $A B J C$ và $A N J Q$ là các hình bình hành nên $J B|A C, J Q| A N$ dẫn đến $J, Q, B$ thẳng hàng. Tương tự $J, P, C$ thẳng hàng.

Gọi $H$ là hình chiếu của $A$ lên $B C$ thì tứ giác $A Q B H$ nội tiếp.

Suy ra

$\quad\quad\quad\quad\quad\quad \angle Q H B=\angle Q A B=\angle Q A M=\angle Q P M=\angle Q P I$

nên tứ giác $P Q H I$ cũng nội tiếp. Gọi $(O)$ là đường tròn ngoại tiếp tam giác $A B C$ thì dễ thấy đường tròn $(A H I)$ tiếp xúc với $(O)$ tại $A$.

Xét ba đường tròn $(O),(A H I),(P Q H I)$ thì

  • Trục đẳng phương của $(O),(A H I)$ là tiếp tuyến của $(O)$ tại $A$.
  • Trục đẳng phương của $(O),(P Q H I)$ là $P Q$.
  • Trục đẳng phương của $(P Q H I),(A H I)$ là $H I$.

Do đó, $K$ chính là tâm đẳng phương của ba đường tròn nên $K \in H I$ hay $K, B, C$ thẳng hàng.

Bài 8. Tìm số nguyên dương $n$ lớn nhất thỏa mãn các điều kiện sau:

$\quad (i)$ n không chia hết cho 3 .

$\quad (ii)$ Bảng vuông $n \times n$ không thể được phủ kín bằng 1 quân tetramino $1 \times 4$ và các quân trimino $1 \times 3$. Trong phép phủ, các quân tetramino và trimino được phép quay dọc nhưng không được phép chườm lên nhau hoặc nằm ra ngoài bảng vuông.

Lời giải. Ta sẽ chứng minh $n=5$ là giá trị lớn nhất cần tìm.

Ta nhận thấy rằng nếu $n=3 k+1, k \geq 1$ thì ta luôn phủ được bảng vuông $n \times n$ bằng cách phủ hàng đầu tiên bằng 1 quân tetramino kích thước $1 \times 4$ (ta sẽ gọi tắt là tetramino) và $k-1$ quân trimino kích thước $1 \times 3$ (ta sẽ gọi tắt là trimino). Các cột còn lại có chiều dài $3 k$ có thể phủ được bằng các quân trimino (xoay dọc lại).

Ta chứng minh rằng nếu $n=3 k+2, k \geq 2$ thì bảng vuông $n \times n$ cũng phủ được. Cách phủ với $n=8$ được minh họa như sau

Dễ dàng thấy rằng với $k \geq 3$ thì ta có thể thu được cách phủ cho bảng vuông $n \times n$ bằng cách phủ phần hình vuông $8 \times 8$ ở góc trên bên trái như trên, phần còn lại gồm 1 hình chữ nhật kích thước $3(k-2) \times(3 k+2)$ và 1 hình chữ nhật kích thước $8 \times 3(k-2)$ phủ được bằng các quân trimino.

Bây giờ ta chứng minh bảng vuông $5 \times 5$ không thể phủ được bằng 1 quân tetramino và 7 quân trimino.

Trước hết ta chứng minh bổ đề: Nếu bảng vuông $5 \times 5$ có thể phủ được bằng một hình vuông $1 \times 1$, ta gọi là unomino và 8 quân trimino thì quân unomino $1 \times 1$ phải phủ ô trung tâm.

Thật vậy,

Ta đánh số các ô của bảng vuông $5 \times 5$ như hình vẽ

Ta thấy rằng một quân trimino luôn phủ đúng một ô mang số 1 , một ô mang số 2 và một ô mang số 3 . Vì số các số 2 bằng 9 , còn số các số 1 và 3 bằng 8 nên nếu phép phủ ở đề bài thực hiện được thì quân unomino phải phủ một ô mang số 2 .

Mặt khác, ta có thể đánh số bảng vuông $5 \times 5$ bằng một cách khác

Các tính chất nói ở trên vẫn đúng cho cách đánh số này, tuy nhiên ở đây số số 1 là 9 , còn số số 2 và 3 là 8 . Do đó, một lần nữa ta kết luận quân unomino phải phủ một ô mang số 1 .

Giao hai điều kiện cần nói trên lại, ta thấy với một cách phủ hợp lệ thì quân unomino phải phủ ô trung tâm.

Quay trở lại với vấn đề phủ bảng vuông $5 \times 5$ bằng 1 quân tetramino và 7 quân trimino. Nếu tồn tại một cách phủ như thế thì cắt quân tetramino thành 1 quân unomino và 1 quân trimino, ta thu được một phép phủ bảng vuông $5 \times 5$ bằng 1 quân unomino và 8 quân trimino.

Theo bổ đề thì quân unomino phải nằm ở ô trung tâm, nghĩa là một đầu của quân tetramino phải nằm ở ô trung tâm, mâu thuẫn (vì khi đó quân tetramino sẽ bị lòi ra ngoài bảng vuông).

Với những lý luận ở trên, ta kết luận $n=5$ là giá trị lớn nhất cần tìm.

 

 

 

 

 

 

 

 

 

 

 

 

 

 

 

 

 

 

 

 

 

 

 

 

 

 

 

 

 

 

 

 

 

 

 

 

 

 

 

 

 

 

 

 

 

 

 

 

 

 

 

 

 

 

 

 

 

 

 

 

 

 

 

 

 

 

 

 

 

 

 

 

 

 

 

 

 

 

 

 

 

 

 

 

 

Đề thi và đáp án kì thi chọn đội tuyển thi Quốc gia trường Phổ thông Năng khiếu năm học 2011 – 2012

 

ĐỀ THI

Ngày thi thứ nhất

Bài 1. Cho các số $a, b, c>0$ thoả mãn $a b+b c+c a=1$. Chứng minh rằng:

$\quad\quad\quad\quad\quad\quad\quad\quad\frac{1}{3+2\left(a^2-b c\right)}+\frac{1}{3+2\left(b^2-c a\right)}+\frac{1}{3+2\left(c^2-a b\right)} \geq 1$

Bài 2. Có bao nhiêu bộ số nguyên dương $(x, y, z, t)$ thoả mãn

$\quad\quad\quad\quad\quad\quad\quad\quad\quad\quad 12<x<y<z<t \text { và } x+y+z+t=2011 ?$

Bài 3. Cho tam giác $A B C$ nội tiếp đường tròn $(O, R)$. Gọi $\left(\mathcal{C}_1\right)$ là đường tròn thay đổi luôn qua $B, C$ và lần lượt cắt các cạnh $A B, A C$ tại $M, N$ khác $B, C$.

(a) Chứng minh rằng $(A M N)$ luôn tiếp xúc với một đường cố định.

(b) Cho $B, C$ cố định, $B C=2 R$ và $A$ thay đổi trên $(O)$. Đường thẳng qua $A$ vuông góc $B C$ cắt $(O)$ tại $D$ và cắt $\left(\mathcal{C}_1\right)$ tại $E, F$. Chứng minh rằng nếu $A$ và $\left(\mathcal{C}_1\right)$ thay đổi sao cho $\frac{E F}{A D}=\frac{\sqrt{5}}{2}$ thì $(A M N)$ luôn tiếp xúc với một đường cố định.

Bài 4. Cho $p$ là số nguyên tố lẻ và đa thức $Q(x)=(p-1) x^p-x-1$. Xét dãy số $\left(a_n\right)$ thoả mãn

$\quad\quad\quad\quad\quad\quad\quad\quad a_0=\frac{p-1}{2}, a_n=a_{n-1}+Q\left(a_{n-1}\right) \forall n \in \mathbb{N}^* .$

(a) Chứng minh rằng với mọi số nguyên dương $n$ thì $\left(a_n, p\right)=1$.

(b) Chứng minh rằng với mọi số nguyên dương $n$ thì $Q\left(a_n\right) \equiv 0\left(\bmod p^n\right)$.

Ngày thi thứ hai

Bài 5. Cho dãy số $\left(u_n\right)$ thoả mãn $u_1=\frac{1}{6}$ và $u_{n+1}=u_n^2+\frac{2}{3} u_n \forall n \in \mathbb{N}^*$.

Tìm $\lim \frac{5 u_{n+1}^2-2 u_n^2 u_{n+1}+5 u_n u_{n+1}}{3 u_n^2+u_n u_{n+1}\left(4+u_n^2\right)}$.

Bài 6. Cho hàm số $f: \mathbb{N} \times \mathbb{N} \rightarrow \mathbb{N}$ thoả mãn $f(0,0)=0$ và:

$\quad\quad\quad\quad\quad\quad\quad\quad f(a, b)=\left\{\begin{array}{l}f\left(\left\lfloor\frac{a}{2}\right\rfloor,\left\lfloor\frac{b}{2}\right\rfloor\right) \text { khi } a+b \equiv 0 \quad(\bmod 2) \\ 1+f\left(\left\lfloor\frac{a}{2}\right\rfloor,\left\lfloor\frac{b}{2}\right\rfloor\right) \text { khi } a+b \equiv 1 \quad(\bmod 2)\end{array}\right.$

(a) Có bao nhiêu số tự nhiên $m \leq 2011$ sao cho $f(2011, m)=5$ ?

(b) Cho số lẻ $p$, cho $n \in \mathbb{N}\left(1<p<2^n\right)$ và $A$ là tập hợp gồm $p$ số tự nhiên không vượt quá $2^n-1$. Chứng minh rằng $\sum_{{a, b} \subset A} f(a, b) \leq n \cdot \frac{p^2-1}{4}$.

Bài 7. Cho tam giác $A B C$ nội tiếp đường tròn $(O)$ với $B, C$ cố định và $A$ thay đổi trên $(O)$. Đường trung trực $d$ của $B C$ cắt $A B, A C$ tại $M, N$. Gọi $P, Q$ lần lượt là các điểm đối xứng của $M, N$ qua $O . K$ là giao điểm của $B P$ và $C Q$.

(a) Chứng minh rằng $K$ luôn thuộc một đường tròn cố định.

(b) Kết luận trên còn đúng không khi $d$ là đường thẳng Euler của tam giác $A B C ?$

Bài 8. Với mọi số nguyên dương $n$, đặt $S_n=x^n+y^n+z^n$. Ta đã biết rằng $S_n=$ $P_n(s, t, p)$ với $s=x+y+z, t=x y+y z+z x, p=x y z$. Hãy tính tổng các hệ số của các đơn thức chứa $p$ trong $P_{2011}(s, t, p)$.

 

LỜI GIẢI

Bài 1. Cho các số $a, b, c>0$ thoả mãn $a b+b c+c a=1$. Chứng minh rằng:

$\quad\quad\quad\quad\quad\quad\quad\quad \frac{1}{3+2\left(a^2-b c\right)}+\frac{1}{3+2\left(b^2-c a\right)}+\frac{1}{3+2\left(c^2-a b\right)} \geq 1$

Lời giải. Đặt $a b=x ; b c=y ; c a=z$ thì ta có $x+y+z=1$. Khi đó áp dụng bất đẳng thức Cauchy-Schwarz, ta có

$\quad\quad\quad\quad\quad \sum_{c y c} \frac{1}{3+2\left(\frac{x z}{y}-y\right)} =\sum_{c y c} \frac{y^2}{3 y^2+2 x y z-2 y^3} $

$\quad\quad\quad\quad\quad\quad\quad\quad\quad\quad\quad\quad  \geq \frac{1}{3\left(x^2+y^2+z^2\right)+6 x y z-2\left(x^3+y^3+z^3\right)} .$

Ta đưa về chứng minh

$\quad\quad\quad\quad\quad\quad\quad\quad 3\left(x^2+y^2+z^2\right)+6 x y z-2\left(x^3+y^3+z^3\right) \leq 1 .$

Tuy nhiên đây lại là đẳng thức vì

$\quad\quad\quad\quad 3\left(x^2+y^2+z^2\right)+6 x y z-2\left(x^3+y^3+z^3\right) $

$\quad\quad\quad\quad\quad =3\left(x^2+y^2+z^2\right)-2(x+y+z)\left(x^2+y^2+z^2-x y-y z-z x\right) $

$\quad\quad\quad\quad\quad =3\left(x^2+y^2+z^2\right)-2\left(x^2+y^2+z^2\right)+2(x y+y z+z x) $

$\quad\quad\quad\quad\quad =(x+y+z)^2=1$

Đẳng thức xảy ra khi $x=y=z=1$ hay $a=b=c=\frac{1}{\sqrt{3}}$.

Bài 2. Có bao nhiêu bộ số nguyên dương $(x, y, z, t)$ thoả mãn $12<x<y<z<t$ và $x+y+z+t=2011 ?$

Lời giải. Đặt $x^{\prime}=x-12 ; y^{\prime}=y-12 ; z^{\prime}=z-12 ; t^{\prime}=t-12$. Phương trình đã cho tương đương với:

$\quad\quad\quad\quad\quad x^{\prime}+y^{\prime}+z^{\prime}+t^{\prime}=2011-48=1963 \text { với } 0 \leq x^{\prime}<y^{\prime}<z^{\prime}<t^{\prime}.$

Theo bài toán chia kẹo Euler thì nếu không có điều kiện thứ hai, số nghiệm của phương trình trên sẽ là $C_{1966}^3$. Ta sẽ trừ ra các trường hợp các số bị trùng nhau

  • Số bộ có 3 số giống nhau là $C_4^3 \cdot\left(1+\left\lfloor\frac{1963}{3}\right\rfloor\right)=2620=A$.
  • Số bộ có 2 số giống nhau là $C_4^2\left(\sum_{a=0}^{981}(1964-2 a)\right)=5791836=B$.

Do mỗi bộ nghiệm như trên chỉ tồn tại 1 cách sắp xếp $x, y, z, t$ thỏa mãn nên số bộ thoả mãn đề bài là

$\quad\quad\quad\quad\quad\quad\quad\quad \frac{C_{1966}^3-B+2 A}{4 !}=\frac{C_{1966}^3-5786596}{4 !} .$

Bài 3. Cho tam giác $A B C$ nội tiếp đường tròn $(O, R)$. Gọi $\left(\mathcal{C}_1\right)$ là đường tròn thay đổi luôn qua $B, C$ và lần lượt cắt các cạnh $A B, A C$ tại $M, N$ khác $B, C$.

(a) Chứng minh rằng $(A M N)$ luôn tiếp xúc với một đường cố định.

(b) Cho $B, C$ cố định, $B C=2 R$ và $A$ thay đổi trên $(O)$. Đường thẳng qua $A$ vuông góc $B C$ cắt $(O)$ tại $D$ và cắt $\left(\mathcal{C}_1\right)$ tại $E, F$. Chứng minh rằng nếu $A$ và $\left(\mathcal{C}_1\right)$ thay đổi sao cho $\frac{E F}{A D}=\frac{\sqrt{5}}{2}$ thì $(A M N)$ luôn tiếp xúc với một đường cố định.

Lời giải. (a) Gọi $d$ là đường thẳng qua $A$, song song với $B C$ và cắt đường tròn $(O)$ tại $T$. Bằng biến đổi góc, ta có

$\quad\quad\quad\quad\quad\quad\quad\quad \angle T A C=\angle A C B=\angle A M N .$

Suy ra $A T$ là tiếp tuyến của đường tròn $(A M N)$ nên ( $A M N)$ tiếp xúc với đường thẳng $d$ cố định.

(b) Gọi $H$ là hình chiếu của $A$ lên $B C$. Xét phương tích từ $A$ đến $\mathcal{C}_1$ thì

$\quad\quad\quad\quad\quad\quad\quad\quad A M \cdot A B=A N \cdot A C=A E \cdot A F \text {. }$

Ta có $H E \cdot H F=H B \cdot H C=H A^2$, mà $\frac{E F}{A D}=\frac{\sqrt{5}}{2}$ nên $H E+H F=\sqrt{5} A H$. Giải hệ này, ta có

$\quad\quad\quad\quad\quad\quad\quad\quad H E=\frac{\sqrt{5}-1}{2} A H \text { và } H F=\frac{\sqrt{5}+1}{2} A H .$

Suy ra $A E=A H-H E=\frac{3-\sqrt{5}}{2} A H$ và $A F=A H+H F=\frac{3+\sqrt{5}}{2} A H$. Từ đó ta được

$\quad\quad\quad\quad\quad\quad\quad\quad A E \cdot A F=\frac{3-\sqrt{5}}{2} A H \cdot \frac{3+\sqrt{5}}{2} A H=A H^2 \text {. }$

Vì thế nên $A H^2=A M \cdot A B=A N \cdot A C$, chứng tỏ $H M, H N$ lần lượt vuông góc với $A B, A C$. Suy ra $(A M N)$ có đường kính là $A H$ nên $(A M N)$ tiếp xúc với $B C$ là đường thẳng cố định.

Bài 4. Cho $p$ là số nguyên tố lẻ và đa thức $Q(x)=(p-1) x^p-x-1$. Xét dãy số $\left(a_n\right)$ thoả mãn

$\quad\quad\quad\quad\quad\quad\quad\quad a_0=\frac{p-1}{2}, a_n=a_{n-1}+Q\left(a_{n-1}\right) \forall n \in \mathbb{N}^* .$

(a) Chứng minh rằng với mọi số nguyên dương $n$ thì $\operatorname{gcd}\left(a_n, p\right)=1$.

(b) Chứng minh rằng với mọi số nguyên dương $n$ thì $Q\left(a_n\right) \equiv 0\left(\bmod p^n\right)$.

Lời giải. (a) Ta có

$\quad\quad\quad\quad a_0=\frac{p-1}{2} \text { và } a_1=a_0+(p-1) a_0^p-a_0-1=\frac{(p-1)^{p+1}}{2^p}-1$

không chia hết cho $p$.

Giả sử tồn tại $k$ nhỏ nhất sao cho $p \mid a_k$ thì $k \geq 2$. Ta có

$\quad\quad\quad\quad a_k=(p-1) a_{k-1}^p-1 \text { và } p-1 \equiv-1, a_{k-1}^p \equiv a_k \quad(\bmod p) .$

Suy ra $a_{k-1} \equiv-1(\bmod p)$ từ đó ta được $a_{k-2} \equiv 0(\bmod p)$, mâu thuẫn với tính nhỏ nhất của $k$. Vậy nên ta phải có $\operatorname{gcd}\left(a_n, p\right)=1$ với mọi $n$ nguyên dương.

(b) Ta có $Q(x)=(p-1) x^p-x-1 \equiv(-1) x-x-1=-2 x-1(\bmod p)$ với mọi $x$ nguyên nên

$\quad\quad\quad\quad Q\left(a_1\right) \equiv-2 a_1-1 \equiv-2(p-1) \frac{p-1}{2}+2-1=0 \quad(\bmod p)$

nên khẳng định đúng với $n=1$. Ta sẽ chứng minh bằng quy nạp.

$\quad\quad\quad\quad Q\left(a_{n+1}\right)=(p-1)\left(a_{n+1}^p-a_n^p\right)=(p-1) Q\left(a_n\right)\left(\sum_{i=1}^p a_n^{i-1} a_{n+1}^{p-i}\right).$

Giả sử rằng $p^n \mid Q\left(a_n\right)$ nên suy ra

$\quad\quad\quad\quad a_{n+1} \equiv a_n \quad(\bmod p) \Rightarrow \sum_{i=1}^p a_n^{i-1} a_{n+1}^{p-i} \equiv p a_n^{p-1} \equiv 0 \quad(\bmod p) .$

Như vậy $p^{n+1} \mid Q\left(a_{n+1}\right)$.Theo nguyên lí quy nạp thì ta có điều phải chứng minh.

Bài 5. Cho dãy số $\left(u_n\right)$ thoả mãn $u_1=\frac{1}{6}$ và $u_{n+1}=u_n^2+\frac{2}{3} u_n \forall n \in \mathbb{N}^*$.

Tìm $\lim \frac{5 u_{n+1}^2-2 u_n^2 u_{n+1}+5 u_n u_{n+1}}{3 u_n^2+u_n u_{n+1}\left(4+u_n^2\right)}$.

Lời giải. Trước hết, ta sẽ tìm giới hạn của dãy $\left(u_n\right)$. Bằng quy nạp, ta sẽ chứng minh rằng $0<u_n<\frac{1}{3}, \forall n$. Thật vậy,

  • Với $n=1$ thì khẳng định đúng.
  • Giả sử khẳng định đúng với $n=k>1$ thì $0<u_k<\frac{1}{3}$. Ta có:

$\quad\quad\quad\quad\quad\quad\quad\quad 0<u_{k+1}=u_k^2+\frac{2}{3} u_k<\frac{1}{9}+\frac{2}{3} \cdot \frac{1}{3}=\frac{1}{3}$

nên khẳng định cũng đúng với $n=k+1$.

Theo nguyên lí quy nạp, khẳng định được chứng minh. Xét hàm số $f(x)=x^2+$ $\frac{2}{3} x, x \in\left(0 ; \frac{2}{3}\right)$ thì $f^{\prime}(x)=2 x+\frac{2}{3}>0$ nên đây là hàm đồng biến. Dãy số đã cho chính là $u_1=\frac{1}{6}, u_{n+1}=f\left(u_n\right), n=1,2,3, \ldots$

Hơn nữa $u_2=\frac{1}{6^2}+\frac{2}{3} \cdot \frac{1}{6}=\frac{5}{36}<\frac{1}{6}$ nên đây là dãy giảm và bị chặn dưới nên có giới hạn. Gọi $l$ là giới hạn của dãy thì

$\quad\quad\quad\quad\quad\quad\quad\quad l=l^2+\frac{2}{3} l \Leftrightarrow l=0 \text { hay } l=\frac{1}{3} \text {. }$

Nhưng do dãy này giảm và theo chứng minh trên thì $0<u_n<\frac{1}{3}, \forall n$ nên giới hạn của dãy là 0 .

Theo công thức xác định dãy, ta có $\frac{u_{n+1}}{u_n}=u_n+\frac{2}{3}$. Do dãy $\lim u_n=0$ nên dãy tương ứng $\left(\frac{u_{n+1}}{u_n}\right)$ có giới hạn là $\frac{2}{3}$. Từ đó, ta tính được

$\quad\quad\quad\quad\quad\quad\quad\quad\quad\quad\quad\quad\quad\quad =\frac{5\left(\frac{2}{3}\right)^2-2 \cdot 0+5 \cdot \frac{2}{3}}{3+\frac{2}{3}\left(4+0^2\right)}=\frac{50}{51} .$

Bài 6 . Cho hàm số $f: \mathbb{N} \times \mathbb{N} \rightarrow \mathbb{N}$ thoả mãn $f(0,0)=0$ và:

$\quad\quad\quad\quad\quad\quad\quad\quad f(a, b)=\left\{\begin{array}{l}f\left(\left\lfloor\frac{a}{2}\right\rfloor,\left\lfloor\frac{b}{2}\right\rfloor\right) \text { khi } a+b \equiv 0 \quad(\bmod 2) \\ 1+f\left(\left\lfloor\frac{a}{2}\right\rfloor,\left\lfloor\frac{b}{2}\right\rfloor\right) \text { khi } a+b \equiv 1 \quad(\bmod 2)\end{array}\right.$

(a) Có bao nhiêu số tự nhiên $m \leq 2011$ sao cho $f(2011, m)=5$ ?

(b) Cho số lẻ $p$, cho $n \in \mathbb{N}\left(1<p<2^n\right)$ và $A$ là tập hợp gồm $p$ số tự nhiên không vượt quá $2^n-1$. Chứng minh rằng $\sum_{{a, b} \subset A} f(a, b) \leq n \cdot \frac{p^2-1}{4}$.

Lời giải. (a) Đổi số 2011 sang hệ nhị phân, ta có $2011=\overline{11111011011}(2)$. Khi đổi số $m$ sang hệ nhị phân, ta cũng có tương ứng $m=\overline{a_1 a_2 \ldots a_{11}(2)}$ (do $m \leq 11$ nên ta chỉ xét 11 chữ số).

Do công thức xác định của hàm, ta thấy $f(2011, m)$ chính bằng số vị trí trong dãy chữ số trên mà hai chữ số tại cùng vị trí là khác tính chẵn lẻ.

Trong 11 chữ số của $m$, ta chọn 5 vị trí để cho chúng khác tính chẵn lẻ với các chữ số của 2011 thì có $C_{11}^5=462$ cách. Mỗi cách chọn tính chẵn lẻ đó tương ứng với đúng một số $m$.

Tuy nhiên, ta phải trừ đi trường hợp đổi tính chẵn lẻ tại vị trí thứ 6 (và giữ nguyên từ $a_1 \rightarrow a_5$ ), tức là

$\quad\quad\quad\quad\quad\quad\quad\quad a_1=a_2=\cdots=a_6=1$

khi đó thì $m>2011$, không thỏa mãn. Ta sẽ đếm số cách chọn $m$ như thế. Trong 5 vị trí từ $a_7 \rightarrow a_{11}$, chọn ra 4 vị trí để đổi tính chẵn lẻ, có $C_5^4=5$ cách. Chú ý rằng số 0 ở vị trí thứ 9 không ảnh hưởng vì sau nó chỉ còn 2 vị trí, không đủ để thực hiện chọn ra 4 vị trí để đổi tính chẵn lẻ như trên.

Vậy nên số các số $m$ thỏa mãn là $462-5=457$.

(b) Đổi tất cả $p$ số của tập $A$ sang hệ nhị phân thì mỗi số sẽ có không quá $n$ chữ số và xếp vào bảng ô vuông kích thước $p \times n$. Mỗi dòng tương ứng với một số, và số nào không có đủ $n$ chữ số trong hệ nhị phân thì ta thêm 0 vào trước nó. Khi đó, tổng $\sum_{{a, b} \subset A} f(a, b)$ chính bằng tổng các cặp vị trí khác nhau trên mỗi cột.

$\quad\quad\quad\quad\quad\quad\quad\quad\quad\quad\quad\quad \begin{array}{|l|l|l|l|}\hline a_{1,1} & a_{1,2} & \cdots & a_{1, n} \\ \hline a_{2,1} & a_{2,2} & \cdots & a_{2, n} \\ \hline \cdots & \cdots & \cdots & \cdots \\ \hline a_{p, 1} & a_{p, 2} & \cdots & a_{p, n} \\ \hline\end{array}$

Xét cột thứ 1 , giả sử trên đó có $x$ số 0 và $y$ số 1 với $x+y=p$. Khi đó, số cặp chữ số khác nhau trên cột này sẽ là

$\quad\quad\quad\quad x y=\frac{1}{4}\left[(x+y)^2-(x-y)^2\right]=\frac{1}{4}\left[p^2-(x-y)^2\right] \leq \frac{p^2-1}{4}$

(do $x, y$ khác tính chẵn lẻ nên $|x-y| \geq 1$ ). Tương tự với các cột khác, số cặp chữ số khác nhau cũng không vượt quá $\frac{p^2-1}{4}$. Và do tính độc lập giữa các cột, ta có

$\quad\quad\quad\quad\quad\quad\quad\quad \sum_{{a, b} \subset A} f(a, b) \leq n \cdot \frac{p^2-1}{4} .$

Bài 7. Cho tam giác $A B C$ nội tiếp đường tròn $(O)$ với $B, C$ cố định và $A$ thay đổi trên $(O)$. Đường trung trực $d$ của $B C$ cắt $A B, A C$ tại $M, N$. Gọi $P, Q$ lần lượt là các điểm đối xứng của $M, N$ qua $O$. $K$ là giao điểm của $B P$ và $C Q$.

(a) Chứng minh rằng $K$ luôn thuộc một đường tròn cố định.

(b) Kết luận trên còn đúng không khi $d$ là đường thẳng Euler của tam giác $A B C ?$

Lời giải. Ta sẽ chứng minh bài toán tổng quát khi thay trung trực $B C$ lẫn đường thẳng Euler bởi đường thẳng $d$ bất kỳ đi qua $O$.

Kẻ đường kính $B B^{\prime}, C C^{\prime}$ của $(O)$ và giả sử $B^{\prime} N, C^{\prime} M$ cắt nhau ở $T$. Khi đó, vì $M, N, O$ thẳng hàng nên theo định lý Pascal đảo thì lục giác tạo bởi các đỉnh $A, B, C, B^{\prime}, C^{\prime}, T$ nội tiếp. Do đó, $T \in(O)$.

Ngoài ra, vì $\angle B T N=\angle B T B^{\prime}=90^{\circ}$ nên $T B \perp T N$, tương tự thì $T C \perp T M$.

Kẻ đường kính $T K^{\prime}$ của $(O)$ thì do $O$ là trung diểm chung của $T K^{\prime}, M P$ nên tứ giác $T M K^{\prime} P$ là hình bình hành. Suy ra $T M | K^{\prime} P$ nên $K^{\prime} P \perp T C$.

Mà tứ giác $C M C^{\prime} P$ cũng là hình bình hành nên $C P | C^{\prime} M$, mà $C^{\prime} M \perp T C$ nên $C P \perp T C$. Từ các điều này, ta suy ra $K^{\prime}, P, C$ thẳng hàng. Tương tự thì $K^{\prime}, Q, B$ thẳng hàng. Vì thế nên $K^{\prime} \equiv K$, hay $K$ luôn thuộc đường tròn $(O)$ cố định.

Nhận xét. Trong bài toán trên, $T$ chính là giao điểm của hai đường tròn đường kính $B N, C M$. Nếu gọi $S$ là giao điểm còn lại thì ta chứng minh được bằng phép nghịch đảo trực tâm $H$ rằng $S$ nằm trên đường tròn Euler của tam giác $A B C$.

Bài 8. Với mọi số nguyên dương $n$, đặt $S_n=x^n+y^n+z^n$. Ta đã biết rằng $S_n=P_n(s, t, p)$ với $s=x+y+z, t=x y+y z+z x, p=x y z$. Hãy tính tổng các hệ số của các đơn thức chứa $p$ trong $P_{2011}(s, t, p)$.

Lời giải. Theo định lý Viete thì $x, y, z$ là nghiệm của phương trình

$\quad\quad\quad\quad\quad\quad\quad\quad\quad a^3-s a^2+t a-p=0 .$

Để tính tổng hệ số của tất cả các đơn thức trong $P_{2011}$, ta xét $P_{2011}(1,1,1)$. Tương tự, tổng các hệ số của các đơn thức không chứa $p$ trong $P_{2011}$ là $P_{2011}(1,1,0)$. Do đó, ta cần tính

$\quad\quad\quad\quad\quad\quad\quad\quad M=P_{2011}(1,1,1)-P_{2011}(1,1,0) .$

Xét phương trình $a^3-a^2+a-1=0$ có ba nghiệm là $a=1, a=i$ và $a=-i$. Vì $P_{2011}(s, t, p)=x^n+y^n+z^n$ nên ta có

$\quad\quad\quad\quad\quad\quad\quad P(1,1,1)=1^{2011}+i^{2011}+(-i)^{2011}=1 .$

Tiếp tục xét $a^3-a^2+a=0$ có ba nghiệm là $a=0, a=\frac{1 \pm i \sqrt{3}}{2}$. Áp dụng công thức Moivre của lũy thừa số phức, ta tính được

$\quad\quad\quad\quad P(1,1,0) =0^{2011}+\left(\frac{1+i \sqrt{3}}{2}\right)^{2011}+\left(\frac{1-i \sqrt{3}}{2}\right)^{2011} $

$\quad\quad\quad\quad\quad\quad\quad\quad =\left(\cos \frac{\pi}{3}+i \sin \frac{\pi}{3}\right)^{2011}-\left(\cos \frac{2 \pi}{3}+i \sin \frac{2 \pi}{3}\right)^{2011} $

$\quad\quad\quad\quad\quad\quad\quad\quad =\left(\cos \frac{2011 \pi}{3}+i \sin \frac{2011 \pi}{3}\right)-\left(\cos \frac{4022 \pi}{3}+i \sin \frac{4022 \pi}{3}\right)=1$

Vì thế nên $M=0$.

 

 

 

 

 

 

 

 

 

 

 

 

 

 

 

 

 

 

 

 

 

 

 

 

 

 

 

 

 

 

 

 

 

 

 

 

 

 

 

 

 

 

 

 

 

 

 

 

 

 

 

 

 

 

 

 

 

 

 

 

 

Đề thi và đáp án kì thi chọn đội tuyển thi Quốc gia trường Phổ thông Năng khiếu năm học 2009 – 2010

ĐỀ THI

Ngày thi thứ nhất

Bài 1. Cho $a, b, c$ là các số thực để đa thức $P(x)=x^4+ax^3+b x^2+c x+1$ có ít nhất một nghiệm thực. Tìm tất cả các bộ $(a, b, c)$ để $a^2+b^2+c^2$ đạt giá trị nhỏ nhất.

Bài 2. Cho $A={1,2, \ldots, 2 n}$. Một tập con của $A$ được gọi là tốt nếu như có đúng 2 phần tử $x, y$ và đồng thời $|x-y| \in[1, n]$. Tìm số các tập hợp $[A_1, A_2, \ldots, A_n]$ để $A_i$ là tập con tốt của $A$ với $1 \leq i \leq n$ và $\bigcup_{i=1}^n A_i=A$.

Bài 3. Tìm tất cả các hàm số $f: \mathbb{N}^* \rightarrow \mathbb{N}^*$ thoả mãn các điều kiện sau:

$\quad\quad(i) f $ là hàm số tăng thật sự trên $\mathbb{N}^*$.

$\quad\quad(ii) f(f(n))=4 n+9 \forall n \in \mathbb{N}^*$.

$\quad\quad(iii) f(f(n)-n)=2 n+9 \forall n \in \mathbb{N}^*$.

Bài 4. Cho đường tròn tâm $O$ và dây cung $A B$ cố định khác đường kính. Một điểm $P$ thay đổi trên cung lớn $A B$. Gọi $I$ là trung điểm của $A B$. Lấy các điểm $M, N$ trên các tia $P A, P B$ sao cho $\angle P M I=\angle P N I=\angle A P B$.

(a) Chứng minh rằng đường cao từ $P$ của tam giác $P M N$ luôn đi qua một điểm cố định.

(b) Chứng minh rằng đường thẳng Euler của tam giác $P M N$ luôn đi qua một điểm cố định.

Ngày thi thứ hai

Bài 5. Cho $a, b, c$ là các số thực dương. Giải hệ phương trình sau:

$\quad\quad\quad\quad\quad\quad\quad\quad\quad\quad\left\{\begin{array}{l}a x-a b y+\frac{1}{x y}=b c^2 \\ a b z-b c^2 x+\frac{1}{x z}=a . \\ b c^2 y-a z+\frac{1}{y z}=a b\end{array}\right.$

Bài 6. Cho dãy số $\left(a_n\right)$ xác định bởi $a_1=a, a_{n+1}=\left(a_1+\cdots+a_n-2\right)^2 \forall n \in \mathbb{N}^*$. Đặt $S_n=a_1+a_2+\cdots+a_n$. Tìm tất cả các giá trị $a$ để dãy số $\left(S_n\right)$ hội tụ.

Bài 7. Tìm tất cả các số nguyên dương $k$ để phương trình sau có nghiệm nguyên dương $(x, y)$ :

$$\quad\quad x^2+y^2+x+y=k x y$

Bài 8. Cho tam giác $A B C$ nội tiếp đường tròn $(O)$. Gọi $I, I_1, I_2, I_3$ lần lượt là tâm đường tròn nội tiếp và tâm đường tròn bàng tiếp các đỉnh $A, B, C$ của tam giác $A B C$. Dường tròn ngoại tiếp tam giác $I I_2 I_3$ cắt $(O)$ tại hai điểm $M_1, N_1$. Gọi $J_1$ là giao điểm của $A I$ và $(O)$. Ký hiệu $d_1$ là đường thẳng qua $J_1$ và vuông góc với $M_1 N_1$. Xác định các đường thẳng $d_2, d_3$ tương tự. Chứng minh rằng $d_1, d_2, d_3$ dồng quy.

 

LỜI GIẢI

Ngày thi thứ nhất

Bài 1. Cho $a, b, c$ là các số thực để đa thức $P(x)=x^4+a x+3+b x^2+c x+1$ có ít nhất một nghiệm thực. Tìm tất cả các bộ $(a, b, c)$ để $a^2+b^2+c^2$ đạt giá trị nhỏ nhất.

Lời giải. Gọi $x_0$ là một nghiệm của $P(x)$ (dễ thấy $x_0 \neq 0$ ). Do $P\left(x_0\right)=0$ nên ta có

$\quad\quad\quad\quad\quad\quad\quad\quad\quad\quad-\left(x_0^4+1\right)=a x_0^3+b x_0^2+c x_0 .$

Sử dụng bất đẳng thức Cauchy-Schwarz, ta có

$\quad\quad\quad\quad\quad\left(x_0^4+1\right)^2=\left(a x_0^3+b x_0^2+c x_0\right)^2 \leq\left(a^2+b^2+c^2\right)\left(x_0^6+x_0^4+x_0^2\right) .$

Đặt $t=x_0^2>0$. Từ đánh giá trên, ta suy ra

$\quad\quad\quad\quad\quad\quad\quad\quad\quad\quad a^2+b^2+c^2 \geq \frac{\left(t^2+1\right)^2}{t^3+t^2+t}=\frac{\left(t^2+1\right)^2}{t\left(t^2+t+1\right)}$

Mà theo bất đẳng thức AM-GM thì

$\quad\quad\quad\quad\quad\quad\quad\quad\quad\quad t \leq \frac{t^2+1}{2} \text { và } t^2+t+1 \leq t^2+\frac{t^2+1}{2}=\frac{3}{2}\left(t^2+1\right) .$

Do đó

$\quad\quad\quad\quad\quad\quad\quad\quad\quad\quad\frac{\left(t^2+1\right)^2}{t\left(t^2+t+1\right)} \geq \frac{4}{3}, \text { nên } a^2+b^2+c^2 \geq \frac{4}{3}$

Đẳng thức xảy ra khi và chỉ khi

$\quad\quad\quad\quad\quad\quad\quad\quad\quad\quad\left\{\begin{array}{l}x_0^4+a x_0^3+b x_0^2+c x_0+1=0 \\ x_0^2=1 \\ \frac{a}{x_0^3}=\frac{b}{x_0^2}=\frac{c}{x_0}\end{array}\right.$

Giải hệ này, ta thu được $a=b=c=-\frac{2}{3}$ hoặc $a=-b=c=\frac{2}{3}$.

Bài 2. Cho $A=[1,2, \ldots, 2 n]$. Một tập con của $A$ được gọi là tốt nếu như có đúng 2 phần tử $x, y$ và đồng thời $|x-y| \in[1, n]$. Tìm số các tập hợp $[A_1, A_2, \ldots, A_n]$ để $A_i$ là tập con tốt của $A$ với $1 \leq i \leq n$ và $\bigcup_{i=1}^n A_i=A$.

Lời giải . Gọi $u_n, n \in{1,2, \ldots, n}$ là số các tập hợp $[A_1, A_2, \ldots, A_n]$ thỏa mãn yêu cầu đề bài, đồng thời hai phần tử $n$ và $n+1$ không đi cùng nhau trong bất kì tập $A_i$ nào. Ta chia các số $1,2, \ldots, 2 n$ vào một bảng $2 \times n$ như sau

$\quad\quad\quad\quad\quad\quad\quad\quad\quad\quad\quad\quad\quad\quad\begin{array}{|c|c|c|c|}\hline 1 & 2 & \ldots & n \\ \hline n+1 & n+2 & \ldots & 2 n \\ \hline\end{array}$

Khi đó, mỗi cách chọn được liệt kê trong $u_n$ tương ứng với một cách chọn từ bảng trên các cặp gồm hai số ở cùng một cột hoặc hai số liên tiếp nhau trên cùng một hàng. Xét $u_{n+1}$, vì phần tử $2(n+1)$ chỉ có thể đi cùng với $n+1$ hoặc $2 n+1$ trong cùng một tập $A_i$ nào đó nên ta xét hai khả năng sau.

  • $2(n+1)$ và $n+1$ cùng thuộc một tập $A_i$, giả sử là $A_{n+1}$.

$\quad\quad\quad\quad\quad\quad\quad\quad\quad\quad\quad\quad\begin{array}{|c|c|l|c|}\hline 1 & 2 & \ldots & n+1 \\ \hline n+1 & n+2 & \ldots & 2(n+1) \\ \hline\end{array}$

Lúc này, mỗi cách chọn một bộ $[A_1, A_2, \ldots, A_n]$ ứng với một cách chọn các cặp số gồm các số ở cùng một cột hoặc ở cạnh nhau trong cùng một hàng từ một bảng $2 \times n$. Theo định nghĩa của ta số cách chọn như thế là $u_n$. Vậy trong trường hợp này có $u_n$ cách chọn.

  • $2(n+1)$ và $2 n+1$ cùng thuộc một tập $A_i$, giả sử đó là $A_{n+1}$.

$\quad\quad\quad\quad\quad\quad\quad\quad\quad\quad\quad\quad\begin{array}{|c|c|l|c|c|}\hline 1 & 2 & \ldots & n & n+1 \\ \hline n+1 & n+2 & \ldots & 2 n+1 & 2(n+1) \\ \hline\end{array}$

Ta thấy $n+1$ chỉ có thể đi cùng với $2(n+1)$ (trường hợp $n+1$ đi cùng với $n+2$ không được xét trong $\left.u_{n+1}\right)$ nhưng $2(n+1)$ đã đi cùng với $2 n+1$ nên $n+1$ phải đi cùng với $n$ trong cùng một tập $A_i$ nào đó, giả sử là $A_n$. Lập luận tương tự trường hợp trên, ta suy ra số cách chọn các tập ${A_1, A_2, \ldots, A_n-1}$ là $u_{n-1}$.

Theo quy tắc cộng, ta có $u_{n+1}=u_n+u_{n-1}$. Mặt khác, $u_1=1$ và $u_2=2$ nên ta tìm được công thức tổng quát của $u_n$ là

$\quad\quad\quad\quad\quad\quad\quad\quad\quad\quad\quad\quad u_n=\frac{1}{\sqrt{5}}\left[\left(\frac{1+\sqrt{5}}{2}\right)^{n+1}-\left(\frac{1-\sqrt{5}}{2}\right)^{n+1}\right]$

Xét trường hợp sinh ra bộ $[A_1, A_2, \ldots, A_n]$ có $n$ và $n+1$ đi cùng trong một tập $A_i$ nào đó.

$\quad\quad\quad\quad\quad\quad\quad\quad\quad\quad\quad\quad\begin{array}{|c|c|l|c|}\hline 1 & 2 & \ldots & n \\ \hline n+1 & n+2 & \ldots & 2 n \\ \hline\end{array}$

Rõ ràng 1 chỉ có thể đi cùng với 2 hoặc $n+1$ nhưng $n+1$ đã đi cùng $n$ nên 1 chỉ có thể đi cùng với 2 . Tiếp theo, $n+2$ có thể đi cùng với $2, n+1$ hay $n+3$ nhưng 2 đã đi với 1 còn $n+1$ đã đi với $n$ nên $n+2$ phải đi với $n+3$.

Tiếp tục, 3 có thể đi cùng 2,4 hay $n+3$ nhưng 2 đã đi với 1 còn $n+3$ đã đi với $n+2$ nên 3 phải đi với 4 . Tiếp tục lý luận như trên, ta suy ra $A_i$ phải có dạng

$\quad\quad\quad [1,2],[3,4], \ldots,[n-2, n-1],[n, n+1],[n+1, n+2], \ldots,[2 n-1,2 n]$

Từ đó suy ra trường hợp này chỉ cho ta duy nhất một bộ $[A_1, A_2, \ldots A_n]$ nếu $n$ lẻ và không có bộ nào nếu $n$ chẵn.

Vậy số các bộ $[A_1, A_2, \ldots A_n]$ thỏa mãn đề bài là

$\quad\quad\quad\quad\quad\quad\quad\quad\begin{cases}\frac{1}{\sqrt{5}}\left[\left(\frac{1+\sqrt{5}}{2}\right)^{n+1}-\left(\frac{1-\sqrt{5}}{2}\right)^{n+1}\right] & , n=1,n \text { chẳn. } \\ 1+\frac{1}{\sqrt{5}}\left[\left(\frac{1+\sqrt{5}}{2}\right)^{n+1}-\left(\frac{1-\sqrt{5}}{2}\right)^{n+1}\right] & , n>1, n \text { lẻ’ }\end{cases}$

Bài 3. Tìm tất cả các hàm số $f: \mathbb{N}^* \rightarrow \mathbb{N}^*$ thoả mãn các điều kiện sau:

$\quad\quad(i) f $ là hàm số tăng thật sự trên $\mathbb{N}^*$.

$\quad\quad(ii) f(f(n))=4 n+9 \forall n \in \mathbb{N}^*$.

$\quad\quad(iii) f(f(n)-n)=2 n+9 \forall n \in \mathbb{N}^*$.

Lời giải. Vì $f: \mathbb{N}^* \longrightarrow \mathbb{N}^*$ tăng ngặt nên

$\quad\quad\quad\quad\quad\quad\quad\quad\quad\quad f(a)-f(b) \geq a-b, \forall a, b \in \mathbb{N}^*, a>b .$

Theo điều kiện (iii), ta có

$2=2(n+1)+9-(2 n+9) =f(f(n+1)-(n+1))-f(f(n)-n) $

$\quad\quad\quad\quad\quad\quad\quad\quad\quad\quad\quad\quad \geq f(n+1)-(n+1)-[f(n)-n] $

$\quad\quad\quad\quad\quad\quad\quad\quad\quad\quad\quad\quad =f(n+1)-f(n)-1 .$

Do đó $f(n+1)-f(n) \leq 3$

với mọi $n \in \mathbb{N}^*$, tức $f(n+1)-f(n) \in\ {1,2,3}$

với mọi $n \in \mathbb{N}^*$. Ta xét các trường hợp sau

  • Giả sử tồn tại $n \in \mathbb{N}^*$ sao cho $f(n+1)-f(n)=1$ thì

$\quad\quad\quad\quad\quad\quad\quad\quad\quad\quad f(n+1)-(n+1)=f(n)-n,$

suy ra $2(n+1)+9=f(f(n+1)-(n+1))=f(f(n)-n))=2 n+9$, vô lí.

  • Giả sử tồn tại $n \in \mathbb{N}^*$ sao cho $f(n+1)-f(n)=3$ thì

$\quad\quad\quad\quad\quad\quad\quad\quad\quad\quad f(n+1)-(n+1)=f(n)-n+2$

Ta lại có

$\quad\quad\quad\quad\quad\quad\quad\quad\quad\quad f(f(n+1)-(n+1))-f(f(n)-n)=2$

Do đó, nếu $f(n)>n$ thì đặt $t=f(n)-n \in \mathbb{N}^*$, ta suy ra $f(t+2)=t+2$.

Mà $f$ tăng ngặt nên $f(t+1)-f(t)=1$, mâu thuẫn.

Vậy $f(n) \leq n$ với mọi $n \in \mathbb{N}^*$ suy ra

$f(n)=n$ với mọi $n \in \mathbb{N}^*$.

Khi đó ta lại có $n=f(n)=f(f(n))=4 n+9,$ tức $n=-3$, vô lí.

Như vậy, $f(n+1)-f(n) \notin{1,3}$ với mọi $n \in \mathbb{N}^*$, nên

$\quad\quad\quad\quad\quad\quad\quad\quad\quad\quad f(n+1)-f(n)=2 \text {, với mọi } n \in \mathbb{N}^* \text {. }$

Ta suy ra $f(n)=2 n+k$. Thay vào đề bài, ta thu được $k=3$. Vậy $f(n)=2 n+3$ là nghiệm duy nhất của phương trình.

Bài 4. Cho đường tròn tâm $O$ và dây cung $A B$ cố định khác đường kính. Một điểm $P$ thay đổi trên cung lớn $A B$. Gọi $I$ là trung điểm của $A B$. Lấy các điểm $M, N$ trên các tia $P A, P B$ sao cho $\angle P M I=\angle P N I=\angle A P B$.

(a) Chứng minh rằng đường cao từ $P$ của tam giác $P M N$ luôn đi qua một điểm cố định.

(b) Chứng minh rằng đường thẳng Euler của tam giác $P M N$ luôn đi qua một điểm cố định.

Lời giải. (a) Kí hiệu $X=M I \cap P B, Y=N I \cap P A$. Ta có $\angle P M I=\angle P N I=$ $\angle A P B$ nên các tam giác $P M X$ và $P N Y$ cân tại $X, Y$. Từ đó suy ra

$\angle P X M=\angle P Y N=180^{\circ}-2 \angle A P B,$

suy ra $M, N, X, Y$ đồng viên. Gọi $S$ là tâm đường tròn ngoại tiếp tam giác $A O B$ thì $S$ cố định. Ta có $\angle I S B=180^{\circ}-\angle A O B=180^{\circ}-2 \angle A P B=\angle P X M$. Tương tự, ta suy ra $\angle I S A=\angle P Y N$. Do đó $I, S, X, B$ đồng viên và $I, S, Y, A$ đồng viên. Suy ra

$\angle S X B=\angle S Y A=\angle S I B=90^{\circ} .$

Suy ra $I S$ là đường kính của đường tròn ngoại tiếp tam giác $P X Y$. Mặt khác, $M, N, X, Y$ dồng viên nên nên $M N$ và $X Y$ đối song nhau trong $\angle A P B$, tức $I S \perp$ $M N$. Nói cách khác, đường cao từ $P$ của tam giác $P M N$ đi qua điểm $S$ cố định.

(b) Trước tiên, ta chứng minh bổ đề sau.

BỔ ĐỀ. Cho tam giác $A B C$ và đường tròn $(\omega)$ đi qua hai điểm $B, C$ và cắt các canh $A B, A C$ tại $X, Y$. Gọi $X X^{\prime}, Y Y^{\prime}$ là các đường cao của tam giác $A X Y$. Gọi $B B^{\prime}, C C^{\prime}$ là các đường cao của tam giác $A B C$. Gọi $H, H^{\prime}$ là các trục tâm của tam giác $A B C$ và tam giác $A X Y$. Kí hiệu $I \equiv B Y \cap C X$. Khi đó $H, I, H^{\prime}$ thẳng hàng.

Chứng minh. Ta có $X, Y, X^{\prime}, Y^{\prime}$ đồng viên nên $\overline{H^{\prime} X} \cdot \overline{H^{\prime} X^{\prime}}=\overline{H^{\prime} Y} \cdot \overline{H^{\prime} Y^{\prime}}$, tức là

$P_{H^{\prime} /[B Y]}=P_{H^{\prime} /[C X]},$

trong đó $[U V]$ là đường tròn đường kính $U V$. Ta có $B, C, B^{\prime}, C^{\prime}$ đồng viên nên $\overline{H B} \cdot \overline{H B^{\prime}}=\overline{H C} \cdot \overline{H C^{\prime}}$, tức

$P_{H /[B Y]}=P_{H /[C X]} .$

Cuối cùng $B, C, X, Y$ đồng viên nên $\overline{I B} \cdot \overline{I Y}=\overline{I C} \cdot \overline{I X}$, tức

$P_{I /[B Y]}=P_{I /[C Y]} .$

Suy ra $H, I, H^{\prime}$ thẳng hàng vì cùng thuộc trục đẳng phương của $[B Y]$ và $[C X]$.

Trở lại bài toán,

Gọi $H, O^{\prime}$ lần lượt là trực tâm và tâm đường tròn ngoại tiếp của tam giác $P M N$. Ta có $O^{\prime} P=O^{\prime} M$ và $X P=X M$ nên $X O^{\prime}$ là đường trung trực của $P M$, suy ra $X O^{\prime} \perp P Y$. Tương tự ta cũng có $Y O^{\prime} \perp P X$.

Vì thế nên $O^{\prime}$ cũng chính là trực tâm của tam giác $P X Y$. Áp dụng bổ đề cho tam giác $P X Y$ với $(\omega) \equiv(M N X Y)$ thì ta có $O^{\prime}, H, I \equiv Y N \cap M X$ thẳng hàng. Hay nói cách khác, đường thẳng Euler $O^{\prime} H$ của tam giác $P M N$ đi qua điểm $I$ cố định. Bài toán được giải quyết.

Ngày thi thứ hai

Bài 5. Cho $a, b, c$ là các số thực dương. Giải hệ phương trình sau:

$\quad\quad\quad\quad\quad\quad\quad\quad\quad\quad\left\{\begin{array}{l}a x-a b y+\frac{1}{x y}=b c^2 \\ a b z-b c^2 x+\frac{1}{x z}=a \\ b c^2 y-a z+\frac{1}{y z}=a b\end{array}\right.$

Lời giải. Đặt $(m, n, p)=\left(a, a b, b c^2\right)$. Khi đó $m, n, p>0$. Hệ phương trình trở thành

$\quad\quad\quad\quad\quad\quad\quad\quad\quad\quad\left\{\begin{array}{l}m x-n y+\frac{1}{x y}=p \\ n z-p x+\frac{1}{z x}=m \\ p y-m z+\frac{1}{y z}=n,\end{array}\right.$

tương đương

$\quad\quad\quad\quad\quad\quad\quad\quad\quad\quad\left\{\begin{aligned}m x-n y-p &=-\frac{1}{x y}, \\ -m+n z-p x &=-\frac{1}{z x}, \\ -m z+p y-n &=-\frac{1}{y z} .\end{aligned}\right.$

Xem hệ trên là hệ phương trình tuyến tính theo ẩn $m, n, p$, ta có

$\quad\quad D=\left|\begin{array}{ccc}x & -y & -1 \\ -1 & z & -x \\ -z & -1 & y\end{array}\right|=x y z-x y z-1-x^2-y^2-z^2=-1-\left(x^2+y^2+z^2\right) \neq 0$

$\quad\quad\quad\quad\quad\quad\quad \quad\quad D_m=\left|\begin{array}{ccc}x & -\frac{1}{x y} & -1 \\ -1 & -\frac{1}{z x} & -x \\ -z & -\frac{1}{y z} & y\end{array}\right|=-\frac{1+x^2+y^2+z^2}{z x} .$

Tương tự, ta cũng tính được

$\quad\quad\quad\quad\quad\quad D_n=-\frac{1+x^2+y^2+z^2}{y z} \text {, và } D_p=-\frac{1+x^2+y^2+z^2}{x y} .$

Do đó

$\quad\quad\quad\quad\quad\quad\quad (m, n, p)=\left(\frac{D_m}{D}, \frac{D_n}{D}, \frac{D_p}{D}\right)=\left(\frac{1}{z x}, \frac{1}{y z}, \frac{1}{x y}\right) .$

Thay $(m, n, p)=\left(a, a b, b c^2\right)$, ta được

$\quad\quad\quad\quad\quad\quad\quad\quad\quad\quad x y=\frac{1}{b c^2}, y z=\frac{1}{a b}, z x=\frac{1}{a} .$

Nhân ba phương trình trên vế theo vế rồi lấy căn bậc hai, ta được $x y z=\pm \frac{1}{a b c}$.

  • Với $x y z=\frac{1}{a b c}$, ta suy ra $x=\frac{1}{c}, y=\frac{1}{b c}, z=\frac{c}{a}$.
  • Với $x y z=-\frac{1}{a b c}$, ta suy ra $x=-\frac{1}{c}, y=-\frac{1}{b c}, z=-\frac{c}{a}$.

Vậy hệ có 2 nghiệm là $\left(\frac{1}{c}, \frac{1}{b c}, \frac{c}{a}\right)$ và $\left(-\frac{1}{c},-\frac{1}{b c},-\frac{c}{a}\right)$.

Bài 6. Cho dãy số $\left(a_n\right)$ xác định bởi $a_1=a, a_{n+1}=\left(a_1+\cdots+a_n-2\right)^2 \forall n \in \mathbb{N}^*$.

Đặt $S_n=a_1+a_2+\cdots+a_n$. Tìm tất cả các giá trị $a$ để dãy số $\left(S_n\right)$ hội tụ.

Lời giải. Từ giả thiết suy ra $S_{n+1}-S_n=\left(S_n-2\right)^2$. Do đó dãy $\left(S_n\right)$ được xác định như sau

$\quad\quad\quad\quad\quad\quad\quad\quad\quad\quad\left\{\begin{array}{l}S_1=a, \\ S_{n+1}=f\left(S_n\right)=S_n^2-3 S_n+4 .\end{array}\right.$

Hơn nữa $f^{\prime}(x)=0$ có nghiệm duy nhất $x=\frac{2}{3}$ nên ta có thể vẽ bảng biến thiên, khảo sát được hàm số này. Từ đó, nhờ việc các trường hợp của $a$, ta thấy

  • Nếu $a>2$. Giả sử $[S_n]$ có giới hạn $L$ thì ta phải có $L=f(L)$ nên $L \in{1,2}$. Mà $\left(S_n\right)$ không giảm nên $L \geq a>2$, mâu thuẫn. Vậy nếu $a>2$ thì $\left(S_n\right)$ không hội tụ.
  • Nếu $a<1$ thì suy ra $S_2=f(a)>2$. Quay về trường hợp 1 , ta suy ra $\left(S_n\right)$ không hội tụ.
  • Nếu $1 \leq a \leq 2$ thì từ bảng biến thiên, ta có $\frac{7}{4} \leq S_n \leq 2$ với mọi $n \in \mathbb{N}^*$. Từ đó $\left(S_n\right)$ không giảm và bị chặn nên $\left(S_n\right)$ hội tụ.

Vậy các giá trị của $a$ thỏa mãn đề bài là $a \in[1,2]$.

Bài 7. Tìm tất cả các số nguyên dương $k$ để phương trình sau có nghiệm nguyên dương $(x, y)$ :

$\quad\quad\quad\quad\quad\quad\quad\quad\quad\quad\quad x^2+y^2+x+y=k x y .$

Lời giải. Không mất tính tổng quát, giả sử $x \geq y$. Xét giá trị $k$ sao cho phương trình đã cho có nghiệm nguyên dương. Trong các nghiệm ấy, gọi $\left(x_0, y_0\right)$ là nghiệm sao cho $x_0 \geq y_0 \geq 0$ và $x_0$ nhỏ nhất. Xét tam thức

$\quad\quad\quad\quad\quad\quad\quad\quad\quad f(x)=x^2-\left(k y_0-1\right) x+y_0^2+y_0 .$

Khi đó $f\left(x_0\right)=0$. Theo định lí Viette, $f(x)$ còn một nghiệm khác là $x_0^{\prime}=k y_0-1-x_0$. Tuy nhiên, theo cách chọn $\left(x_0, y_0\right)$ thì ta có $x_0^{\prime} \geq x_0 \geq y_0$ nên $y_0$ nằm ngoài hai khoảng nghiệm của tam thức bậc hai $f(x)$. Mà hệ số cao nhất của $f(x)$ dương nên $f\left(y_0\right) \geq 0$. Do $f\left(y_0\right) \geq 2 y_0^2+2 y_0-k y_0^2$ nên ta có

$\quad\quad\quad\quad\quad\quad\quad\quad\quad\quad\quad k \leq 2+\frac{2}{y_0} \leq 4 .$

Suy ra $k \in{1,2,3,4}$.

  • Nếu $k=1$ thì phương trình có dạng $x^2+y^2+x+y=x y$, tương đương với

$\quad\quad\quad\quad\quad\quad\quad\quad\left(x-\frac{y}{2}\right)^2+\frac{3}{4} y^2+x+y=0$ (vô lí vì $\left.x, y>0\right)$.

  • Nếu $k=2$ thì phương trình có dạng $x^2+y^2+x+y=2 x y$, tương đương với

$\quad\quad\quad\quad\quad\quad\quad\quad\quad\quad(x-y)^2+x+y=0$ (vô lí vì $x, y>0$ ).

  • Nếu $k=3$ thì phương trình có nghiệm $(x, y)=(2,2)$.
  • Nếu $k=4$ thì phương trình có nghiệm $(x, y)=(1,1)$.

Vậy các giá trị cần tìm là $k=3, k=4$.

Bài 8. Cho tam giác $A B C$ nội tiếp đường tròn $(O)$. Gọi $I, I_1, I_2, I_3$ lần lượt là tâm đường tròn nội tiếp và tâm đường tròn bàng tiếp các đỉnh $A, B, C$ của tam giác $A B C$. Đường tròn ngoại tiếp tam giác $I_2 I_3$ cắt $(O)$ tại hai điểm $M_1, N_1$. Gọi $J_1$ là giao điểm của $A I$ và $(O)$. Ký hiệu $d_1$ là đường thẳng qua $J_1$ và vuông góc với $M_1 N_1$. Xác định các đường thẳng $d_2, d_3$ tương tự. Chứng minh rằng $d_1, d_2, d_3$ đồng quy.

Lời giải. Gọi $\left(O^{\prime}\right)$ là đường tròn ngoại tiếp tam giác $I_1 I_2 I_3$ và $\left(O_1\right)$ là đường tròn ngoại tiếp tam giác $\left(I I_2 I_3\right)$. Ta có $A I \perp I_2 I_3, B I \perp I_3 I_1$ nên $I$ là trực tâm tam giác $I_1 I_2 I_3$ và $(O)$ là đường tròn Euler của tam giác $I_1 I_2 I_3$ nên $O$ là trung điểm của $I O^{\prime}$. Mặt khác thì

$\angle I_2 O_1 I_3=2\left(180^{\circ}-\angle I_2 I I_3\right)=2 \angle I_2 I_1 I_3=I_2 O^{\prime} I_3 .$

Do đó $O^{\prime}$ đối xứng với $O_1$ qua $I_2 I_3$, suy ra $\overrightarrow{O^{\prime} O_1}=\overrightarrow{I_1 I}$, tức $A I O_1 O^{\prime}$ là hình bình hành. Mà $O$ là trung điểm $I O^{\prime}$ nên $O$ cũng là trung điểm của $I_1 O_1$. Hơn nữa, $O O_1 \perp M_1 N_1$ (đường nối tâm vuông góc với dây cung) nên $I_1 O \perp M_1 N_1$.

Mặt khác, $J_1$ là trung điểm của $I I_1$ (đường tròn Euler đi qua trung điểm của đoạn thẳng nối trực tâm của tam giác với đỉnh của tam giác ấy) nên phép vị tự tâm $I$, tỉ số $k=\frac{1}{2}$ biến $I_1 O_1$ thành $d_1$. Do đó $d_1$ đi qua trung điểm $S$ của $O I$. Tương tự, ta suy ra $d_2, d_3$ cũng đi qua $S$, tức $d_1, d_2, d_3$ đồng quy.

Nhận xét. Bài toán thực chất là việc đổi mô hình từ một tính chất quen thuộc liên quan đến trực tâm, chân đường cao sang mô hình ba tâm bàng tiếp. Vì thế, đôi khi việc chuyển đổi giữa các mô hình giúp cho bài toán sáng sủa, dễ xử lý hơn.

 

 

 

 

 

 

 

 

 

 

 

 

 

 

 

 

 

 

 

 

 

 

 

 

 

 

 

 

 

 

 

 

 

 

 

 

 

 

 

 

 

 

 

 

 

 

 

 

 

 

 

 

 

 

 

 

 

 

 

 

 

 

 

 

 

 

 

 

 

 

 

 

 

 

 

 

 

 

 

 

 

 

ĐỀ THI VÀO CHUYÊN TOÁN LỚP 10 TP.HCM 2012

Bài 1. Giải phương trình:

$\sqrt{8 x+1}+\sqrt{46-10 x}=-x^{3}+5 x^{2}+4 x+1$

Bài 2. Cho đa thức $f(x)=a x^{3}+b x^{2}+c x+d$ với $a$ là số nguyên dương, biết $f(5)-$ $f(4)=$ 2012. Chứng minh rằng: $f(7)-f(2)$ là hợp số.

Bài 3. Cho ba số dương $a, b, c$ thỏa $a+b+c=1$. Tìm giá trị nhỏ nhất của biểu thức:

$A=14\left(a^{2}+b^{2}+c^{2}\right)+\frac{a b+b c+c a}{a^{2} b+b^{2} c+c^{2} a}$

Bài 4. Cho tứ giác $A B C D$ nội tiếp đường tròn $(O, R)$ có $A C$ vuông góc với $B D$ tại $H$. Trên cạnh $A B$ lấy điểm $M$ sao cho $A B=3 A M$. Trên cạnh $H C$ lấy trung điểm $N$. Chứng minh rằng $M H$ vuông góc với $D N$.

Bài 5. Cho đường tròn tâm $O$ và đường tròn tâm $I$ cắt nhau tại hai điểm $A$ và $B(O$ và $I$ nằm khác phía đối với đường thẳng $A B), I B$ cắt $(O)$ tại $E, O B$ cắt $(I)$ tại $F$. Qua $B$ vẽ đường thẳng $M N$ song song với $E F(M$ thuộc $(O), N$ thuộc $(I)$ ).

(a) Chứng minh rằng $O A I E$ nội tiếp.

(b) Chứng minh rằng: $A E+A F=M N$.

Bài 6. Trên mặt phẳng cho 2013 điểm tùy ý sao cho trong ba điểm bất kì thì tồn tại 2 điểm mà khoảng cách giữa hai điểm đó luôn bé hơn 1. Chứng minh rằng tồn tại một đường tròn có bán kính bằng 1 chứa ít nhất 1007 điểm (kế cả biên).

 

LỜI GIẢI

Bài 1. Giải phương trình:

$\sqrt{8 x+1}+\sqrt{46-10 x}=-x^{3}+5 x^{2}+4 x+1$

Lời giải. $\sqrt{8 x+1}+\sqrt{46-10 x}=-x^{3}+5 x^{2}+4 x+1$

ĐKХĐ: $\frac{-1}{8} \leq x \leq \frac{23}{5}$

Sử dụng lượng liên hợp, phương trình ban đầu tương đương với:

$\sqrt{8 x+1}-3+\sqrt{46-10 x}-6+x^{3}-x^{2}-4 x^{2}+4 x-8 x+8=0$

$\Leftrightarrow(x-1)\left(\frac{8}{\sqrt{8 x+1}+3}-\frac{10}{\sqrt{46-10 x}+6}+x^{2}-4 x-8\right)=0$

Từ đó ta có phương trình có một nghiệm là $x=1$. Xét biểu thức:

$\frac{8}{\sqrt{8 x+1}+3}-\frac{10}{\sqrt{46-10 x}+6}+x^{2}-4 x-8=0$

Từ điều kiện ta có:

$-1<x<5 \Leftrightarrow(x+1)(x-5)<0 \Leftrightarrow x^{2}-4 x-5<0$

Lại có: $\frac{8}{\sqrt{8 x+1}+3} \leq \frac{8}{3}<\frac{9}{3}=3 \Leftrightarrow \frac{8}{\sqrt{8 x+1}+3}-3<0$ Từ đó ta có:

$\frac{8}{\sqrt{8 x+1}+3}-\frac{10}{\sqrt{46-10 x}+6}+x^{2}-4 x-8<0$

Vậy phương trình đã cho có nghiệm duy nhất là: $x=1$

Bài 2. Cho đa thức $f(x)=a x^{3}+b x^{2}+c x+d$ với $a$ là số nguyên dương, biết $f(5)-$ $f(4)=2012$. Chứng minh rằng: $f(7)-f(2)$ là hợp số.

Lời giải. Ta có: $f(x)=a x^{3}+b x^{2}+c x+d$

Từ đó ta tính được: $f(5)=125 a+25 b+5 c+d, f(4)=64 a+16 b+4 c+d$

Vậy: $f(5)-f(4)=61 a+9 b+c=2012, f(7)=343 a+49 b+7 c+d, f(2)=8 a+4 b+$ $2 c+d$

Vậy: $f(7)-f(2)=335 a+45 b+5 c=5(67 a+9 b+c)=30 a+5(61 a+9 b+c)=30 a+$ 10060

Từ đó ta có: $f(7)-f(2)$ là hợp số vì $a$ là số nguyên dương và nó chia hết cho $2,5,10$.

Bài 3. Cho ba số dương $a, b, c$ thỏa $a+b+c=1$. Tìm giá trị nhỏ nhât của biểu thức:

$A=14\left(a^{2}+b^{2}+c^{2}\right)+\frac{a b+b c+c a}{a^{2} b+b^{2} c+c^{2} a}$

Lời giải.

Cách 1:

$\left(a^{2}+b^{2}+c^{2}\right)(a+b+c)=a^{3}+b^{3}+c^{3}+\left(a^{2} b+b^{2} c+c^{2} a\right)+\left(b^{2} a+a^{2} c+c^{2} b\right) $

$\left(a^{2}+b^{2}+c^{2}\right)(a+b+c)=\left(a^{3}+a b^{2}\right)+\left(b^{3}+b c^{2}\right)+\left(c^{3}+c a^{2}\right)+\left(a^{2} b+b^{2} c+c^{2} a\right)$

Áp dụng bất đẳng thức Cauchy và do $a+b+c=1$, ta có:

$\left(a^{2}+b^{2}+c^{2}\right) \geq 2 a^{2} b+2 b^{2} c+2 c^{2} a+\left(a^{2} b+b^{2} c+c^{2} a\right)=3\left(a^{2} b+b^{2} c+c^{2} a\right)$

Mặt khác: $a b+b c+c a=\frac{1-\left(a^{2}+b^{2}+c^{2}\right)}{2}$

Từ đó ta có: $F \geq 14\left(a^{2}+b^{2}+c^{2}\right)+\frac{3-3\left(a^{2}+b^{2}+c^{2}\right)}{2\left(a^{2}+b^{2}+c^{2}\right)}$

Hay: $F \geq 14\left(a^{2}+b^{2}+c^{2}\right)+\frac{3}{2\left(a^{2}+b^{2}+c^{2}\right)}-\frac{3}{2}$

Áp dụng bất đẳng thức Cauchy, ta có:

$27\left(a^{2}+b^{2}+c^{2}\right)+\frac{3}{\left(a^{2}+b^{2}+c^{2}\right)} \geq 2 \sqrt{27\left(a^{2}+b^{2}+c^{2}\right) \cdot \frac{3}{\left(a^{2}+b^{2}+c^{2}\right)}}=18 $

$a^{2}+b^{2}+c^{2} \geq \frac{1}{3}(a+b+c)^{2}=\frac{1}{3}$

Vậy: $28\left(a^{2}+b^{2}+c^{2}\right)+\frac{3}{\left(a^{2}+b^{2}+c^{2}\right)} \geq 18+\frac{1}{3}=\frac{55}{3}$

Từ đó ta có: $F \geq \frac{55}{6}-\frac{3}{2}=\frac{23}{3}$

Đẳng thức xảy ra khi: $a=b=c=\frac{1}{3}$

Cách 2:

Do $a, b, c$ dương và $a+b+c=1$ nên ta có:

$(1-c)^{2}=(a+b)^{2} \geq 4 a b \Leftrightarrow 1-2 c+c^{2} \geq 4 a b \Leftrightarrow a-2 a c+a c^{2} \geq 4 a^{2} b $

$(1-a)^{2}=(b+c)^{2} \geq 4 b c \Leftrightarrow 1-2 a+a^{2} \geq 4 b c \Leftrightarrow b-2 a b+a^{2} b \geq 4 b^{2} c $

$(1-b)^{2}=(c+a)^{2} \geq 4 c a \Leftrightarrow 1-2 b+b^{2} \geq 4 c a \Leftrightarrow c-2 b c+b^{2} c \geq 4 a c^{2}$

Hay: $a+b+c-2(a b+b c+c a) \geq 3\left(a^{2} b+b^{2} c+c^{2} a\right)$

$\Leftrightarrow 1-2(a b+b c+c a) \geq 3\left(a^{2} b+b^{2} c+c^{2} a\right)$

Vậy: $F \geq 14[1-2(a b+b c+c a)]+\frac{3(a b+b c+c a)}{1-2(a b+b c+c a)}$

Đạt: $t=1-2(a b+b c+c a), t \geq \frac{1}{3}$

Áp dụng bất đẳng thức Cauchy ta có:

$F \geq 14 t+\frac{\frac{3}{2}(1-t)}{t}=14 t+\frac{3}{2 t}-\frac{3}{2}=\frac{1}{2} t+\frac{27}{2} t+\frac{3}{2 t}-\frac{3}{2} \geq \frac{1}{2} t+2 \sqrt{\frac{27}{2} t \cdot \frac{3}{2 t}}-\frac{3}{2}$

Vậy: $F \geq \frac{1}{2} \cdot \frac{1}{3}+9-\frac{3}{2}=\frac{23}{3}$

Đẳng thức xảy ra khi: $a=b=c=\frac{1}{3}$

Bài 4. Cho tứ giác $A B C D$ nội tiếp đường tròn $(O, R)$ có $A C$ vuông góc với $B D$ tại $H$. Trên cạnh $A B$ lấy điểm $M$ sao cho $A B=3 A M$. Trên cạnh $H C$ lấy trung điểm $N$. Chứng minh rằng $M H$ vuông góc với $D N$.

Lời giải.

  • Gọi $K, L$ lần lượt là trung điểm $B M$ và $H B, P$ là giao điểm của $H M$ và $A K$.

  • Ta có $K L$ là đường trung bình của tam giác $H M B$ nên $K L$ song song $H M$. Khi đó xét tam giác $A K L$ thì $P H$ là đường trung bình nên $P$ là trung điểm của $A K$.

  • Ta có từ $A B C D$ nội tiếp suy ra $H D \cdot H B=H A \cdot A C \Rightarrow H K \cdot H D=H A \cdot H N$, do đó $A D N K$ nội tiếp.

  • Suy ra $\angle N H Q=\angle A H P=\angle H A P=\angle H D N$, suy ra $\angle H Q N=90^{\circ}$.

Bài 5. Cho đường tròn tâm $O$ và đường tròn tâm $I$ cắt nhau tại hai điểm $A$ và $B(O$ và $I$ nằm khác phía đối với đường thẳng $A B), I B$ cắt $(O)$ tại $E, O B$ cắt $(I)$ tại $F$. Qua $B$ vẽ đường thẳng $M N$ song song với $E F(M$ thuộc $(O), N$ thuộc $(I))$.

a) Chứng minh rằng $O A I E$ nội tiếp.

b) Chứng minh rằng: $A E+A F=M N$.

Lời giải.

a) Chứng minh rằng tứ giác $A O E F$ nội tiếp

Do hai đường tròn $(\mathrm{O})$ và $(\mathrm{I})$ cắt nhau tại $A$ và $B$ nên ta có: $A$ đối xứng với $B$ qua $O I$. Vậy: $\angle O A I=\angle O B I$

Ta có tam giác $\triangle O B E$ cân tại $O$ nên $\angle O B E=\angle O E B$, do $\angle O B E+\angle O B I=180^{\circ}$ nên $\angle O E B+\angle O B I=180^{\circ}$. Từ đó ta có: $\angle O E B+\angle O A I=180^{\circ}$

Vậy tứ giác $O A I E$ là tứ giác nội tiếp. Chứng minh tương tự ta có: tứ giác $O A I F$ là tứ giác nội tiếp.

$\angle O E A=\angle O I A$ (tứ giác $O A I E$ là tứ giác nội tiếp)

$\angle O I A=\angle O F A$ (tứ giác $O A I F$ là tứ giác nội tiếp)

Vậy: $\angle O E A=\angle O F A$ nên tứ giác $O A F E$ là tứ giác nội tiếp

b) Chứng minh rằng: $M N=A E+A F$

Bài toán cần chứng minh tương đương với: $A F=B N$ và $A E=B M$.

Ta chỉ cần chứng minh $A F=B N$ vì $A E=B M$ là điều tương tự.

Để chứng minh $A F=B N$. Ta chỉ cần chứng minh số đo cung $\mathrm{AF}$ bằng số đo cung $\mathrm{BN}(A F, B N$ lần lượt là dây căng cung $\mathrm{AF}$, cung $\mathrm{BN}$ trong đường tròn (I)). Hay chỉ cần chứng minh: số đo cung $\mathrm{AB}$ bằng số đo cung FN. Từ đó ta chứng minh: $\angle O F A=\angle F B N$ là bài toán được giải quyết.

Do $E F | M N$ nên ta có: $\angle O F E=\angle F B N$

Mà $\angle O F E=\angle O A E=\angle O E A=\angle O F A$ (tứ giác $A O E F$ là tứ giác nội tiếp)

Từ đó ta có: $\angle O F A=\angle F B N$ (đpcm)

Bài 6. Trên mặt phẳng cho 2013 điểm tùy ý sao cho trong ba điểm bất kì thì tồn tại 2 điểm mà khoảng cách giữa hai điểm đó luôn bé hơn 1. Chứng minh rằng tồn tại một đường tròn có bán kính bằng 1 chứa ít nhất 1007 điểm (kế cả biên).

Lời giải. Gọi $A$ là một điểm bất kì trong 2013 điểm trên. Lấy $A$ làm tâm vẽ đường tròn có bán kính bằng 1 .

Nếu 2012 điểm còn lại thuộc đường tròn $(A)$ thì bài toán được chứng minh xong. Giả tồn tại một số điểm nằm ngoài đường tròn tâm $(A)$. Lấy điểm $(B)$ bất kì trong các điểm đó và vẽ đường tròn tâm $(B)$ có bán kính bằng 1 .

Giả sử tồn tại một điểm $C$ nằm ngoài hai đường tròn $(A)$ và $(B)$ thì $A B, A C$ đều lớn hơn 1. Điều này vô lí.

Từ đó ta có tất cả các điểm đã cho đều thuộc trong hai đường tròn $(A)$ và $(B)$.

Theo nguyên lí Dirichlet sẽ tồn tại một đường tròn chứa $\frac{2012}{2}+1=1007$ điểm (đpcm).

 

 

 

 

 

 

 

 

 

 

 

 

 

 

 

 

 

 

 

 

 

 

 

 

 

 

 

 

 

 

 

 

 

 

 

 

 

 

 

 

 

 

 

 

 

 

 

 

 

 

 

ĐỀ THI VÀO LỚP 10 CHUYÊN TOÁN TP.HCM 2013

Bài 1. (a) Giải phương trình: $x \sqrt{2 x-2}+5 x=9$.

(b) Cho $x, y, z$ đôi một khác nhau thỏa mãn: $\frac{1}{x}+\frac{1}{y}+\frac{1}{z}=0$. Tính giá trị biểu thực:

$P=\frac{y z}{x^{2}+2 y z}+\frac{z x}{y^{2}+2 z x}+\frac{x y}{z^{2}+2 x y}$

Bài 2. Cho phương trình $x^{2}-5 m x-4 m=0$.

(a) Định $m$ để phương trình có hai nghiệm phân biệt.

(b) Gọi $x_{1}, x_{2}$ là hai nghiệm của phương trình. Tìm $m$ để biểu thức sau đạt giá trị nhỏ nhất:

$\frac{m^{2}}{x_{1}^{2}+5 m x_{2}+12 m}+\frac{x_{2}^{2}+5 m x_{1}+12 m}{m^{2}}$

Bài 3. Cho tam giác $\triangle A B C$ có $B C$ là cạnh dài nhất. Trên $B C$ lấy hai điểm $D$ và $E$ sao cho $B D=B A, C E=C A$. Đường thẳng qua $D$ song song với $A B$ cắt $A C$ tại $M$. Đường thẳng qua $E$ song song với $A C$ cắt $A B$ tại $N$. Chứng minh rằng $A M=A N$.

Bài 4. Cho $x, y$ là hai số dương thỏa mãn: $x+y=1$. Chứng minh: $3(3 x-2)^{2}+\frac{8 x}{y} \geq$ $7 .$

Bài 5. Từ một điểm $A$ bên ngoài đường tròn $(O)$ vẽ các tiếp tuyến $A B, A C$ và cát tuyến $A E F$ (EF không đi qua $O, B$ và $C$ là các tiếp điểm). Gọi $D$ là điểm đôi xứng của $B$ qua $O . D E, D F$ lần lượt cắt $A O$ tại $M$ và $N$. Chứng minh rằng :

(a) Hai tam giác $\triangle C E F$ và $\triangle C M N$ đồng dạng.

(b) $O M=O N$.

Bài 6. Chữ số hàng đơn vị trong hệ thập phân của số $M=a^{2}+a b+b^{2}$ là $0\left(a ; b \in N^{*}\right)$.

(a) Chứng minh rằng $M$ chia hết cho 20 .

(b) Tìm chữ số hàng chục của $M$.

LỜI GIẢI

Bài 1.

a) Giải phương trình: $x \sqrt{2 x-2}+5 x=9$.

b) Cho $x, y, z$ đôi một khác nhau thỏa mãn: $\frac{1}{x}+\frac{1}{y}+\frac{1}{z}=0$. Tính giá trị biểu thực:

$P=\frac{y z}{x^{2}+2 y z}+\frac{z x}{y^{2}+2 z x}+\frac{x y}{z^{2}+2 x y}$

Lời giải.

a) Giải phương trình: $x \sqrt{2 x-2}+5 x=9$

ĐKXĐ: $x \geq 1$. Đặt $a=\sqrt{2 x-2}$ (ĐKXĐ: $a \geq 0$ )

Phương trình đã cho tương đương với:

$a x=9-5 x=9-\frac{5}{2}\left(a^{2}+2\right)=4-\frac{5}{2} a^{2}$

Ta có hệ phương trình sau:

$\left\{\begin{array} { l }{ 5 a ^ { 2 } + 2 a x = 8 } \\{ a ^ { 2 } – 2 x = – 2 }
\end{array} \Leftrightarrow \left\{\begin{array}{l}
x=\frac{a^{2}+2}{2} \\x=\frac{9}{a+5}
\end{array}\right.\right.$

$\Leftrightarrow \frac{9}{a+5}=\frac{a^{2}+2}{2} \Leftrightarrow a^{3}+5 a^{2}+2 a-8=0 \Leftrightarrow(a-1)(a+2)(a+4)=0$

Kết hợp với: ĐKXĐ: $a \geq 0$. Từ đó ta tính được: $a=1 \Leftrightarrow x=\frac{3}{2}$

b) Tính giá trị biểu thức: $P=\frac{y z}{x^{2}+2 y z}+\frac{z x}{y^{2}+2 z x}+\frac{x y}{z^{2}+2 x y}$

Từ điều kiện của đề bài ta có: $x y+y z+z x=0$

Thêm vào đó: $x^{2}+2 y z=x^{2}+y z-x y-x z=(x-y)(x-z)$

Từ đó ta có:

$P=\sum_{x, y, z} \frac{y z}{x^{2}+2 y z}=\sum_{x, y, z} \frac{y z}{(x-y)(x-z)}=-\frac{y z(y-z)+x z(z-x)+x y(x-y)}{(x-y)(y-z)(z-x)}$

Vậy: $P=1$

Bài 2. Cho phương trình $x^{2}-5 m x-4 m=0$.

a) Định $m$ để phương trình có hai nghiệm phân biệt.

b) Gọi $x_{1}, x_{2}$ là hai nghiệm của phương trình. Tìm $m$ để biểu thức sau đạt giá trị nhỏ nhất:

$\frac{m^{2}}{x_{1}^{2}+5 m x_{2}+12 m}+\frac{x_{2}^{2}+5 m x_{1}+12 m}{m^{2}}$

Lời giải.

a) Định $m$ để phương trình có hai nghiệm phân biệt

ĐKXĐ đề phương trình có hai nghiệm phân biệt là:

$\Delta=(-5 m)^{2}-4(-4 m)=25 m^{2}+16 m=m(25 m+16)>0$

$\Leftrightarrow\left\{\begin{array}{l}m>0 \\ m<\frac{-16}{25}\end{array}\right.$

b) Tìm $m$ để biếu thức sau đạt giá trị nhỏ nhất:

$P=\frac{m^{2}}{x_{1}^{2}+5 m x_{2}+12 m}+\frac{x_{2}^{2}+5 m x_{1}+12 m}{m^{2}}$

Do $x_{1}, x_{2}$ là hai nghiệm của phương trình nên ta có: $\left\{\begin{array}{l}x_{1}^{2}=5 m x_{1}+4 m \\ x_{2}^{2}=5 m x_{2}+4 m\end{array}\right.$

Do phương trình đã cho có hai nghiệm phân biệt nên: $25 m^{2}+16 m>0$. Từ đó áp dụng bất đẳng thức Cauchy, ta có:

$P=\frac{m^{2}}{x_{1}^{2}+5 m x_{2}+12 m}+\frac{x_{2}^{2}+5 m x_{1}+12 m}{m^{2}}$

$P=\frac{m^{2}}{25 m^{2}+16 m}+\frac{25 m^{2}+16 m}{m^{2}} \geq 2$

Đẳng thức xảy ra khi và chỉ khi: $m^{2}=25 m^{2}+16 m \Leftrightarrow m=\frac{-2}{3}$

Bài 3. Cho tam giác $\triangle A B C$ có $B C$ là cạnh dài nhất. Trên $B C$ lấy hai điểm $D$ và $E$ sao cho $B D=B A, C E=C A$. Đường thẳng qua $D$ song song với $A B$ cắt $A C$ tại $M$. Đường thẳng qua $E$ song song với $A C$ cắt $A B$ tại $N$. Chứng minh rằng $A M=A N$.

Lời giải.

Do $D M | A B$, áp dụng định lí Talet:

$\frac{A M}{A C}=\frac{B D}{B C} \Leftrightarrow A M=\frac{B D}{B C} \cdot A C=\frac{B A \cdot A C}{B C}$

Do $E N | A C$, áp dụng định lí Talet:

$\frac{A N}{A B}=\frac{C E}{B C} \Leftrightarrow A N=\frac{C E}{B C} \cdot A B=\frac{B A \cdot A C}{B C}$

Từ đó ta có $A M=A N$. Đây chính là điều phải chứng minh.

Bài 4. Cho $x, y$ là hai số dương thỏa mãn: $x+y=1$. Chứng minh: $3(3 x-2)^{2}+\frac{8 x}{y} \geq 7$.

Lời giải. Do $x+y=1$ nên ta có điều phải chứng minh trở thành:

$3(3 x-2)^{2}+\frac{8 x}{1-x} \geq 7$

Bằng khai triển và biến đổi tương đương ta có: $(5-3 x)(3 x-1)^{2} \geq 0$. Bất đẳng thức này hiển nhiên đúng do $x<1$

Bài 5.Từ một điểm $A$ bên ngoài đường tròn $(O)$ vẽ các tiếp tuyến $A B, A C$ và cát tuyến $A E F$ ( $E F$ không đi qua $O, B$ và $C$ là các tiếp điểm). Gọi $D$ là điểm đối xứng của $B$ qua $O$. $D E, D F$ lần lượt cắt $A O$ tại $M$ và $N$. Chứng minh rằng :

a) Hai tam giác $\triangle C E F$ và $\triangle C M N$ đồng dạng.

b) $O M=O N$.

Lời giải.
a) Chứng minh rằng $\triangle C E F \backsim \triangle C M N$
Ta có: $A N | C D$ (cùng vuông góc với $B C$ )
$\angle D F C=\angle D B C=\angle B A O=\angle C A O$
Từ đó ta có: tứ giác $C F N A$ nội tiếp
Vậy: $\angle C F E=\angle C N M$
Ta có: $A N | C D$ nên: $\angle O M E=\angle C D E$
Do tứ giác $C D F E$ nội tiếp nên: $\angle C D E=\angle C F E$
Vậy: $\angle O M E=\angle C F E$
Mà: $\angle A C E=\angle C F E$ (Tính chất tiếp tuyến)
Từ đó ta có: $\angle A C E=\angle O M E$. Vậy tứ giác $A M E C$ nội tiếp. Nên: $\angle E A M=$ $\angle E C M$

Mà: $\angle E A M=\angle F C N$ (Tứ giác $A N F C$ nội tiếp)

Vậy: $\angle E C M=\angle F C N$

Từ đó ta có: $\angle E C F=\angle M C N$

Do: $\angle C F E=\angle C N M$ và $\angle E C F=\angle M C N$ nên ta có: $\triangle C E F \sim \triangle C M N$

b) Chứng minh rằng: $O M=O N$

Từ giác $A M E C$ nội tiếp: $\angle D C M=\angle C A F$

Từ giác $C F N A$ nội tiếp: $\angle C A F=\angle C N D$

Vậy ta có: $\angle D C M=\angle C N D$ và do: $A N | C D$. Vậy $C D N M$ là hình thang cân nên: $C N=D M$ và $\angle C N M=\angle D M N$

Do $A O$ là đường trung trực của $B C$ nên ta có: $\angle C N M=\angle B N M$ và $N C=N B$

Từ đó ta có: $\angle D M N=\angle B N M$ và $D M=B N$

Hay: $D M | B N$ và $D M=B N$. Từ đó $B M D N$ là hình bình hành. Mà $O$ là trung điểm của $B D$ nên $O$ cũng là trung điểm của $M N$ hay: $O M=O N$ (đpcm)

Bài 6. Chữ số hàng đơn vị trong hệ thập phân của số $M=a^{2}+a b+b^{2}$ là $0\left(a ; b \in N^{*}\right)$.

a) Chứng minh rằng $M$ chia hết cho 20 .

b) Tìm chữ số hàng chục của $M$.

Lới giải.

a) Chứng minh rằng: $M \vdots 20$

Do chữ số hàng đơn vị của $M$ là 0 nên ta có: $M \vdots 5$ và $M \vdots 2$

Giả sử cả $a$ và $b$ đều không chia hết cho 2 . Từ đó ta có:

$\left\{\begin{array} { l }{ a \equiv 1 } \\ { b \equiv 1 }\end{array} \Rightarrow \left\{\begin{array}{l}a^{2} \equiv 1 \\ b^{2} \equiv 1 \\ a b \equiv 1\end{array} \Rightarrow a^{2}+a b+b^{2} \equiv 1 \Rightarrow M \equiv 1(\bmod 2)\right.\right.$

Điều này vô lí: từ đó ta có trong hai số $a$ và $b$ phải có một số chia hết cho 2 .

Giả sử $a \vdots$ 2. Do $M \vdots 2$ nên $b^{2} \vdots 2$. Từ đó ta có: $b \vdots 2$

Vi $a \vdots 2$ và $b \vdots 2$ nên $M \vdots 4$

Do $M \vdots 4$ và $M \vdots 5$ nên ta có: $M \vdots 20$ (đpcm)

b) Nhận xét: Một số chính phương khi chia cho 5 dư 0,1 hoặc 4 .

Ta có $5 \mid a^{2}+a b+b^{2}$, suy ra $5 \mid 4 a^{2}+4 a b+4 b^{2}$ hay $5 \mid(2 a+b)^{2}+3 b^{2}$.

Từ nhận xét trên suy ra $5|b, 5| 2 a+b \Rightarrow 5 \mid a$. Do đó $a^{2}+a b+b^{2}$ chia hết cho $25 .$

Kết hợp với câu a ta có $M$ chia hết cho 100 nên chữ số hàng chục là số 0 .

 

 

 

 

 

 

 

 

 

 

 

 

 

 

ĐỀ THI VÀO LỚP 10 CHUYÊN TOÁN TP.HCM – NĂM 2014

Bài 1. (a) Giải phương trình: $x \sqrt{2 x-3}=3 x-4$

(b) Cho 3 số thực $x, y, z$ thỏa mãn điều kiện: $x+y+z=0 ; x y z \neq 0$. Tính giá trị biểu thức:

$P=\frac{x^{2}}{y^{2}+z^{2}-x^{2}}+\frac{y^{2}}{z^{2}+x^{2}-y^{2}}+\frac{z^{2}}{x^{2}+y^{2}-z^{2}}$

Bài 2. Giải hệ phương trình: $\left\{\begin{array}{l}x+y+\frac{1}{y}=\frac{9}{x} \\ x+y-\frac{4}{x}=\frac{4 y}{x^{2}}\end{array}\right.$

Bài 3. Cho tam giác đều $A B C$ và $M$ là một điểm bất kì trên cạnh $B C$. Gọi $D, E$ lần lượt là hình chiếu vuông góc của $M$ trên $A B$ và $A C$. Xác định vị trí của $M$ để tam giác $M D E$ có chu vi nhỏ nhất.

Bài 4. (a) Cho $x, y$ là 2 số thực khác 0 . Chứng minh rằng: $\frac{x^{2}}{y^{2}}+\frac{y^{2}}{x^{2}} \geq \frac{x}{y}+\frac{y}{x}$

(b) Cho $a, b$ là hai số dương. Tìm giá trị nhỏ nhất của biểu thức: $P=\frac{a^{2}+3 a b+b^{2}}{\sqrt{a b}(a+b)}$

Bài 5. Từ một điểm $M$ nằm ngoài đường tròn $(\mathrm{O})$, kẻ các tiếp tuyến $M A, M B$ với $(\mathrm{O})$ $(A, B$ là các tiếp điểm $)$. Gọi $H$ là giao điểm của $A B$ với $O M, I$ là trung điểm của $M H$. Đường thẳng $A I$ cắt $(\mathrm{O})$ tại điểm $K(K$ khác $A)$.

(a) Chứng minh $H K$ vuông góc với $A I$.

(b) Tính số đo góc $\angle M K B$.

Bài 6. Tìm cặp số nguyên $(x, y)$ thỏa mãn phương trình:

$2015\left(x^{2}+y^{2}\right)-2014(2 x y+1)=25$

LỜI GIẢI

 

Bài 1.

a) Giải phương trình: $x \sqrt{2 x-3}=3 x-4$

b) Cho 3 số thực $x, y, z$ thỏa mãn điều kiện: $x+y+z=0 ; x y z \neq 0$. Tính giá trị biểu thức:

$P=\frac{x^{2}}{y^{2}+z^{2}-x^{2}}+\frac{y^{2}}{z^{2}+x^{2}-y^{2}}+\frac{z^{2}}{x^{2}+y^{2}-z^{2}}$

Lời giải.

a) Giải phương trình: $x \sqrt{2 x-3}=3 x-4 Đ \mathrm{~K} Đ: x \geq \frac{3}{2}$

Phương trình đã cho tương đương với:

$x^{2}(2 x-3)=9 x^{2}-24 x+16 \Leftrightarrow 2 x^{3}-12 x^{2}+24 x-16=0 $

$\Leftrightarrow x^{3}-6 x^{2}+12 x-8=0 \Leftrightarrow(x-2)^{3}=0 \Leftrightarrow x=2$

Ta thấy $x=2$ thỏa yêu cầu bài toán, vậy $x=2$ là nghiệm duy nhất của phương trình.

b) Cho 3 số thực $x, y, z$ thỏa mãn điều kiện: $x+y+z=0 ; x y z \neq 0$. Tính giá trị biểu thức:

$P=\frac{x^{2}}{y^{2}+z^{2}-x^{2}}+\frac{y^{2}}{z^{2}+x^{2}-y^{2}}+\frac{z^{2}}{x^{2}+y^{2}-z^{2}}$

Ta có:

$y+z=-x \Leftrightarrow y^{2}+2 y z+z^{2}=x^{2} \Leftrightarrow y^{2}+z^{2}-x^{2}=-2 y z $

$x+z=-y \Leftrightarrow x^{2}+2 x z+z^{2}=y^{2} \Leftrightarrow x^{2}+z^{2}-y^{2}=-2 x z $

$y+x=-z \Leftrightarrow y^{2}+2 y x+x^{2}=z^{2} \Leftrightarrow y^{2}+x^{2}-z^{2}=-2 y x$

Từ đó ta tính được $P$ :

$P=\frac{x^{2}}{-2 y z}+\frac{y^{2}}{-2 x z}+\frac{z^{2}}{-2 y x}=\frac{x^{3}+y^{3}+z^{3}}{-2 x y z}$

Chú ý:

$x^{3}+y^{3}+z^{3}-3 x y z=0 \Rightarrow x^{3}+y^{3}+z^{3}=3 x y z$

Vậy: $P=\frac{x^{3}+y^{3}+z^{3}}{-2 x y z}=\frac{3 x y z}{-2 x y z}=\frac{-3}{2}$

Bài 2. Giải hệ phương trình: $\left\{\begin{array}{l}x+y+\frac{1}{y}=\frac{9}{x} \\ x+y-\frac{4}{x}=\frac{4 y}{x^{2}}\end{array}\right.$

Lời giải. ĐKXĐ: $x, y \neq 0$

Lấy phương trình (1) trừ phương trình (2) ta thu được:

$\frac{1}{y}+\frac{4}{x}=\frac{9}{x}-\frac{4 y}{x^{2}}  \Leftrightarrow \frac{1}{y}=\frac{5}{x}-\frac{4 y}{x^{2}} \Leftrightarrow x^{2}=5 x y-4 y^{2} \Leftrightarrow x^{2}-5 x y+4 y^{2}=0 $

$\Leftrightarrow(x-4 y)(x-y)=0 \Leftrightarrow\left[\begin{array}{l}x=4 y \\ x=y\end{array}\right.$

Trường hợp 1: $x=4 y$. Thay vào phương trình (1) ta có:

$5 y+\frac{1}{y}=\frac{9}{4 y} \Leftrightarrow 5 y=\frac{5}{4 y} \Leftrightarrow\left[\begin{array} { l }{ y = \frac { 1 } { 2 } } \\ { y = \frac { – 1 } { 2 } }\end{array} \Leftrightarrow \left[\begin{array}{l}x=2, y=\frac{1}{2} \\ x=-2, y=\frac{-1}{2}\end{array}\right.\right.$

Trường hợp $2: x=y$. Thay vào phương trình (1) ta có:

$2 y+\frac{1}{y}=\frac{9}{y} \Leftrightarrow 2 y=\frac{8}{y} \Leftrightarrow\left[\begin{array}{l}y=2 \\ y=-2\end{array} \Leftrightarrow\left[\begin{array}{l}x=2, y=2 \\ x=-2, y=-2\end{array}\right.\right.$

Vậy tập nghiệm của phương trình là: $(x, y)=(2,2),(-2,-2),\left(2, \frac{1}{2}\right),\left(-2, \frac{-1}{2}\right)$

Bài 3. Cho tam giác đều $A B C$ và $M$ là một điểm bất kì trên cạnh $B C$. Gọi $D, E$ lần lượt là hình chiếu vuông góc của $M$ trên $A B$ và $A C$. Xác định vị trí của $M$ để tam giác MDE có chu vi nhỏ nhất.

Lời giải.

  • Gọi độ dài cạnh tam giác đều là $a$.

Ta có $M D \cdot A B+M E \cdot A C=2 S_{A M D}+2 S_{A M C}=2 S_{A B C}$. Hay $(M D+M E)=A H \cdot a$, suy ra $M D+M E=A H$ không đổi.

  • Ta có $D, E$ thuộc đường tròn đường kính $A M$. Vẽ đường kính $D F$, ta có $\angle D F E=$ $\angle D A E=60^{\circ}$.

Suy ra $D E=D F \sin D F E=A M \sin 60^{\circ}$.

$D E$ nhỏ nhất khi và chỉ khi $A M$ nhỏ nhất, khi và chỉ khi $M$ trùng với $H$ trung điểm $B C$.

  • Vậy chu vi tam giác $M D E$ nhỏ nhất khi và chỉ khi $M$ là trung điểm $B C$.

Bài 4.

a) Cho $x, y$ là 2 số thực khác 0 . Chứng minh rằng: $\frac{x^{2}}{y^{2}}+\frac{y^{2}}{x^{2}} \geq \frac{x}{y}+\frac{y}{x}$

b) Cho $a, b$ là hai số dương. Tìm giá trị nhỏ nhất của biểu thức:

$P=\frac{a^{2}+3 a b+b^{2}}{\sqrt{a b}(a+b)}$

Lời giải.

a) Bằng biến đổi tương đương ta có:

$\frac{x^{2}}{y^{2}}+\frac{y^{2}}{x^{2}}-\left(\frac{x}{y}+\frac{y}{x}\right) \geq 0 \Leftrightarrow \frac{(x-y)^{2}\left(\left(x+\frac{1}{2}\right)^{2}+\frac{3}{4} y^{2}\right)}{x^{2} y^{2}} \geq 0$

Bất đẳng thức cuối luôn đúng. Dấu bằng trong bất đẳng thức xảy ra khi $x=y$.

b) Cách 1: Với $a, b$ là hai số dương. Ta có:

$P=\frac{a^{2}+3 a b+b^{2}}{\sqrt{a b}(a+b)}=\frac{(a+b)^{2}+a b}{\sqrt{a b}(a+b)}=\frac{\frac{1}{4}(a+b)^{2}+a b+\frac{3}{4}(a+b)^{2}}{\sqrt{a b}(a+b)} $

$P=\frac{\frac{1}{4}(a+b)^{2}+a b}{\sqrt{a b}(a+b)}+\frac{\frac{3}{4}(a+b)}{\sqrt{a b}}$

Áp dụng bất đẳng thức Cauchy:

$P=\frac{\frac{1}{4}(a+b)^{2}+a b}{\sqrt{a b}(a+b)}+\frac{\frac{3}{4}(a+b)}{\sqrt{a b}} \geq \frac{2 \sqrt{\frac{1}{4} a b(a+b)^{2}}}{\sqrt{a b}(a+b)}+\frac{\frac{3}{4} \cdot 2 \sqrt{a b}}{\sqrt{a b}}=1+\frac{3}{2}=\frac{5}{2}$

Dấu bằng trong bất đẳng thức xảy ra khi $a=b$

Cách 2: Ta có:

$P=\frac{a^{2}+3 a b+b^{2}}{\sqrt{a b}(a+b)}=\frac{(a+b)^{2}+a b}{\sqrt{a b}(a+b)}=\frac{a+b}{\sqrt{a b}}+\frac{\sqrt{a b}}{a+b}=\frac{3}{4} \cdot \frac{a+b}{\sqrt{a b}}+\frac{1}{4} \cdot \frac{a+b}{\sqrt{a b}}+\frac{\sqrt{a b}}{a+b}$

Áp dụng bất đẳng thức Cauchy:

$P \geq \frac{3}{4} \cdot 2+2 \sqrt{\frac{1}{4} \cdot \frac{a+b}{\sqrt{a b}} \cdot \frac{\sqrt{a b}}{a+b}}=\frac{3}{2}+1=\frac{5}{2}$

Dấu bằng trong bất đẳng thức xảy ra khi $a=b$

Bài 5.Từ một điểm $M$ nằm ngoài đường tròn $(\mathrm{O})$, kẻ các tiếp tuyến $M A, M B$ với $(\mathrm{O})$ $(A, B$ là các tiếp điểm $)$. Gọi $H$ là giao điểm của $A B$ với $O M, I$ là trung điểm của $M H$. Đường thẳng $A I$ cắt $(\mathrm{O})$ tại điểm $K(K$ khác $A)$.

a) Chứng minh $H K$ vuông góc với $A I$.

b) Tính số đo góc $\angle M K B$.

Lời giải.

a) Vẽ đường kính $A C, C H$ cắt $A I$ tại $K^{\prime}$.

Dễ thấy hai tam giác $A B C$ và $M H A$ đồng dạng, từ đó suy ra $A C H$ và $M A I$ đồng dạng.

Suy ra $\angle A C H=\angle M A I$, mà $\angle M A I+\angle I A C=90^{\circ}$, suy ra $\angle A C H+\angle I A C=$ $90^{\circ}$.

Do đó $\angle A K^{\prime} C=90^{\circ}$, suy ra $K^{\prime}$ thuộc $(O)$, từ đó $K^{\prime} \equiv K$. Ta có điều cần chứng minh.

b) Ta có $I K \cdot I A=I H^{2}=I M^{2}$.

Suy ra $\triangle I K M \backsim \triangle I M A$, do đó $\angle I M K=\angle I A M=\angle K B H$.

Từ đó tứ giác $B H K M$ nội tiếp, suy ra $\angle B K M=\angle B H M=90^{\circ}$.

Bài 6. Tìm cặp số nguyên $(x, y)$ thỏa mãn phương trình:

$2015\left(x^{2}+y^{2}\right)-2014(2 x y+1)=25$

Lời giải.

Ta có: $2015\left(x^{2}+y^{2}\right)-2014(2 x y+1)=25$

$\Leftrightarrow 2014(x-y)^{2}+x^{2}+y^{2}=2039$

Vậy: $2014(x-y)^{2} \leq 2039 \Leftrightarrow|x-y| \leq 1$

  • Trường hợp 1: $x-y=0$. Ta có: $x^{2}+y^{2}=2039$

Phương trình này không có nghiệm nguyên vì 2039 không chia hết cho $2 .$

  • Trường hợp 2: $x-y=1$. Ta có: $y^{2}+y-12=0$

Phương trình này có nghiệm $y=3$ hay $y=-4$

Từ đó ta có hai cặp nghiệm của phương trình là: $(x, y)={(4 ; 3),(-3 ;-4)}$

  • Trường hợp 3: $x-y=-1$. Ta có: $y^{2}-y-12=0$

Phương trình này có nghiệm $y=-3$ hay $y=4$

Từ đó ta có hai cặp nghiệm của phương trình là: $(x, y)={(-4 ;-3),(3 ; 4)}$

Vậy tập nghiệm của phương trình là: $(x, y)={(4 ; 3),(-3 ;-4),(3 ; 4),(-4 ;-3)}$